You are on page 1of 275

Neonatology

Review: IMAGES
Dara Brodsky, MD
Associate Director of Neonatal Intensive Care Unit
Beth Israel Deaconess Medical Center
Assistant Professor of Pediatrics
Harvard Medical School
Boston, Massachusetts

Camilia R. Martin, MD
Associate Director of Neonatal Intensive Care Unit
Beth Israel Deaconess Medical Center
Assistant Professor of Pediatrics
Harvard Medical School
Boston, Massachusetts
Note to the reader: Although the information in this book has been carefully reviewed for correctness of
dosage and indications, neither the authors nor the publisher can accept any legal responsibility for any
errors or omissions that may be made. Neither the publisher nor the authors make any warranty, expressed
or implied, with respect to the material contained herein. Before prescribing any drug, the reader must
review the manufacturer's current product information (package inserts) for accepted indications, absolute
dosage recommendations, and other information pertinent to the safe and effective use of the product
described.
Neonatology Review: Images
1st edition © 2015. Dara Brodsky and Camilia R. Martin. ISBN #: 978-1-312-77976-1. All rights reserved.
No part of this book may be reproduced, reused, republished, or transmitted in any form, or stored in a data
base or retrieval system, without written permission of the publisher.

With Best Regards


Dr. Alhaytham Sh. Dahdal
MBBS, MRCPCH UK, PABHS, PSCMS
Paediatric Senior Registrar
KSAFHNWR – Tabuk – KSA
For more books
Alhaytham59@yahoo.com
nwafhpaededuc@gmail.com
Preface
Welcome to the first edition of Neonatology Review: Images!
Since publishing Neonatology Review (2003 Hanley & Belfus, 2010 Lulu Press) and Neonatology
Review: Q & A (2012, 2014, Lulu Press) to help fellows and neonatologists study for the Neonatal-
Perinatal Boards, many people have asked us to publish an ebook with images. Thus, we created this
systems-based book that contains short vignettes with radiographs, electrocardiograms, and amplitude-
integrated electroencephalographs. Similar to our other books, the content of this book is based on the
most recent Content Specifications in Neonatal-Perinatal Medicine provided by the American Board of
Pediatrics.
Because this resource is not all-inclusive, we suggest that readers supplement this book with other
materials, such as neonatology textbooks and the on-line journal NeoReviews published by the
American Academy of Pediatrics (AAP). In addition, the AAP electronic publication NeoReviewsPlus
serves as an excellent resource for case-based questions. The board review courses (NeoPREP—
sponsored by the AAP and Specialty Review in Neonatology/Perinatology—sponsored by the Pediatrix
Medical Group) are invaluable. Finally, another excellent additional resource is the Radiology text
Imaging of the Newborn by Kirpalani.
We could not have written this book without receiving images from others. In particular, we would
like to thank Dmitry Dukhovny, MD for his insurmountable efforts to provide us with incredible
images. In addition, we would like to thank the following people for providing additional images: Sara
Bates, MD; Heather Burris, MPH; Jeanne Carroll, MD; Eileen Cowan, MD; Padmaja Itikala, MD; Liza
Konnikova, MD; Ashwini Lakshmanan, MD, MPH; Sarah Morton, MD, PhD; and Eniko Sajti, MD,
PhD.
We wish all the readers of our books lots of success and hope you find this new book helpful.
Brodsky & Martin
I. PULMONOLOGY
Pulmonology Question 1
A female infant born at 36 weeks’ gestation has increased work of breathing
while receiving CPAP with a PEEP of 6 cm H2O and 75% O2. An arterial blood
gas is obtained and the results are: pH=7.25, pCO2=68 mm Hg, pO2=34 mm Hg.
The infant’s anterior-posterior (AP) chest radiograph is shown below.

Of the following, the preferred NEXT step in the management of this infant is:
A. Consultation with the pediatric surgery team
B. Echocardiography
C. Intubation
D. Needle thoracentesis
Pulmonology Answer 1
C. Intubation
The chest radiograph of the infant in this vignette reveals a homogenous
reticulogranular pattern and decreased lung volumes. This is consistent with
surfactant deficiency. The reticulogranular pattern (also known as a ground
glass appearance) represents collapsed alveoli surrounded by inflated alveoli.
Affected infants typically have radiographic findings of air bronchograms, which
are visible because of the surrounding collapsed alveoli. Air bronchograms are
not as apparent on this infant’s chest radiograph.
Pulmonology Question 2
The neonatology team is called to the delivery room of a term female infant
with a prenatal diagnosis of trisomy 21 and bilateral pleural effusions (large on
right, small on left). After birth, the infant is cyanotic with severe respiratory
distress and decreased aeration of the right lung despite positive-pressure
ventilation with 100% FiO2. The infant’s respiratory distress improves after the
neonatology fellow intubates the infant and performs a right-sided needle
thoracentesis, removing 100 mL of chylous fluid. After transfer to the NICU and
placement of an umbilical venous line, an AP chest radiograph is obtained:

After the film, the respiratory therapist pulls the infant’s endotracheal tube
back by 1 cm. Three hours later, the infant has worsening respiratory distress, a
high oxygen requirement, and chest radiograph findings of a moderate to large
right-sided pleural effusion. The neonatology fellow then places a right-sided
chest tube. After placement of the chest tube, the infant’s respiratory distress
persists and the fellow is unable to obtain any fluid from the tube despite
manipulation of the tube. A cross-table lateral radiograph of the infant’s chest is
shown below:
Of the following, the most preferred NEXT step is to:
A. Connect the external chest tube tubing to continuous suction
B. Obtain an AP chest radiograph to confirm that the placement is appropriate
C. Place a chest tube in the left side as well
D. Pull the chest tube back 2 cm to ensure that the distal tip is in the correct
position
Pulmonology Answer 2
B. Obtain an AP chest radiograph to confirm that the placement is appropriate
The infant in this vignette continues to have respiratory distress and
hypoxemia even after a chest tube is placed. A cross-table lateral film
demonstrates whether a chest tube is place posteriorly (preferred location for
draining an effusion) or anteriorly (preferred location for a pneumothorax) but
cannot reveal the tube’s location within the pleural cavity. Thus, an AP chest
radiograph should be obtained for the infant in this vignette to confirm that the
tube is within the pleural cavity. The infant’s follow-up AP chest radiograph
shown below reveals a large right-sided pleural effusion and a chest tube that is
not within the pleural cavity.

The neonatology fellow places a new chest tube and the films below show
that this new tube is in the appropriate position with entry at the 6th intercostal
space with posterior positioning within the pleural cavity.
Pulmonology Question 3
A term male infant is born by stat Cesarean section following prolonged fetal
bradycardia. He emerges through meconium-stained amniotic fluid and requires
bag mask ventilation because of severe respiratory distress and hypoxemia. His
Apgar score is 2 at 1 minute and 4 at 5 minutes of age. He has an abnormal
neurological examination and an abnormal amplitude EEG that prompts
placement of an umbilical venous line and initiation of therapeutic hypothermia.
The infant’s AP chest radiograph is shown below.

Of the following, this infant MOST likely has:


A. A congenital pulmonary airway malformation
B. An intestinal perforation
C. Congenital heart disease
D. Pulmonary hypertension
Pulmonology Answer 3
D. Pulmonary hypertension
The chest radiograph of the infant in this vignette shows a moderate-sized left
tension pneumothorax resulting in a moderately hypoinflated left lung and small
heart size. The right lung reveals patchy areas that are most likely attributable to
meconium aspiration pneumonia. A small right-sided pneumothorax cannot be
excluded. A temperature probe is visible within the esophagus and an umbilical
catheter is appropriately positioned. Infants with meconium aspiration
pneumonia often have secondary surfactant deficiency and pulmonary
hypertension.
Pulmonology Question 4
A 1-day old infant born at 23 weeks’ gestation is currently on HFO with a
mean air pressure of 18 mm Hg and 100% FiO2. After an acute desaturation with
hypotension and electrocardiographic changes on the cardiac monitor, a chest
radiograph is obtained:

Of the following, the most preferred NEXT step in the management of this infant
is:
A. Bilateral needle thoracentesis
B. Immediate transition to synchronized intermittent mandatory ventilation
C. Pericardial thoracentesis for removal of a pericardial effusion
D. Withdrawal of blood from the infant’s umbilical arterial line
Pulmonology Answer 4
D. Withdrawal of blood from the infant’s umbilical arterial line
The clinical presentation and radiographic findings of the infant in this
vignette are most consistent with disseminated intravascular air embolism. The
chest radiograph of this infant shows a lucency in the central chest conforming
to cardiac chambers. There is a tubular lucency on each side of the neck, likely
representing air in vessels.
Affected infants typically present with an acute respiratory deterioration with
cyanosis, hypotension, distant heart sounds, murmur, and an arrhythmia.
Management of intracardiac air involves removal of blood and air from an
umbilical arterial line. Despite this approach, mortality remains extremely high.
Reference: Lee SK, Tanswell AK. Pulmonary vascular air embolism in the
newborn. Arch Dis Child. 1989;64:507-510
Pulmonology Question 5
A 25-week gestation female infant is 4 days of age with the following AP
chest radiograph:

The pathophysiology that is most likely associated with this infant’s lung disease
is (are):
A. A right-sided obstructive cardiac lesion
B. Chorioamnionitis and funisitis
C. Excessive acute fluid overload
D. Prior bilateral pneumothoraces
Pulmonology Answer 5
B. Chorioamnionitis and funisitis
The chest radiograph of the infant in this vignette demonstrates bilateral
interstitial opacities, cystic changes, and hyperinflated lungs. It is possible that
the infant has pneumonia as well as evolving severe lung disease of prematurity.
These radiographic findings are consistent with chronic lung disease of
prematurity but this term cannot be applied as the infant is only 4 days of age.
Premature infants born to women with chorioamnionitis and funisitis may have a
severe inflammatory response leading to radiographic findings of chronic lung
disease in the first week of life.
Pulmonology Question 6
A male infant born at 30 weeks’ gestation has bilateral congenital
chylothoraces that required multiple drainage procedures. He remained
intubated for 3 weeks and then transitioned to CPAP. When the pleural effusions
resolved at 4 weeks of age, the infant started nasogastric breast milk feedings
while on CPAP. An echocardiogram shows that the infant has a structurally
normal heart with a small patent ductus arteriosus. One week later, the infant
has worsening respiratory distress and no signs of infection. The neonatal nurse
practitioner orders a chest radiograph, which is shown below:

Of the following, the most preferred NEXT step in the management of this infant
is:
A. Change to a medium-chain triglyceride formula and begin a trial of
diuretics
B. Consult with a pediatric surgery because of a concern for a pulmonary
sequestration
C. Initiate a course of indomethacin
D. Perform bilateral needle aspirations to treat the recurrent pleural effusions
Pulmonology Answer 6
A. Change to a medium-chain triglyceride formula and begin a trial of diuretics
The chest radiograph in this infant shows diffuse coarse granular densities
without any evidence of pleural effusions. In the setting of a history of chylous
effusions and a worsening respiratory status after initiating breast milk feedings
in a patient with trisomy 21, this might represent dilated lymphatic vessels as
found in congenital pulmonary lymphangiectasia. As a result of this possibility,
the most preferred next step in this infant’s management is to change to a
medium-chain triglyceride formula and begin a trial of diuretics. Potential
therapeutic approaches in the future include immunosuppressive agents,
chemical pleurodesis to induce an inflammatory response and pleural fibrosis, or
surgical approaches such as pleurectomy and pleurodesis or thoracic duct
ligation.
This infant’s chest radiograph does not demonstrate a pulmonary
sequestration or pleural effusions and thus neither surgery nor thoracenteses is
warranted. Before considering treatment of this patient for a patent ductus
arteriosus (PDA), confirmation of a large vessel is required by
echocardiography; even if a large PDA is contributing to this infant’s respiratory
distress, treatment with indomethacin in a patient of this chronological age is
unlikely to be successful.
Reference: Reiterer F, Grossauer K, Morris N, et al. Congenital pulmonary
lymphangiectasis. Paediatr Respir Rev. 2014
Pulmonology Question 7
A 1-day old male infant born at 36 weeks’ gestation has the following chest
radiograph:

Of the following, the most likely diagnosis in this infant is:


A. Bilateral pleural effusions
B. Left-sided pneumothorax only
C. Pneumomediastinum
D. Right and left-sided pneumothoraces
Pulmonology Answer 7
D. Right and left-sided pneumothoraces
The chest radiograph of the infant in this vignette reveals a bilateral
pneumothoraces with collapse of the left lung. Although the film demonstrates a
left chest tube, this tube is not working effectively. There is also a right
pneumothorax but this appears to be small to moderate in size.
Pulmonology Question 8
A full-term female infant has a prenatal diagnosis of left lower extralobar
pulmonary sequestration. A postnatal chest radiograph is obtained:

Of the following, the most accurate statement about this infant is:
A. An additional anomaly is unlikely.
B. The lesion is unlikely to regress.
C. There is a possibility that the sequestration connects with the
gastrointestinal tree.
D. The sequestration most likely communicates with the tracheobronchial
tree.
Pulmonology Answer 8
C. There is a possibility that the sequestration connects with the gastrointestinal
tree
This infant’s chest radiograph shows an opacity in the left lower chest
consistent with a pulmonary sequestration that was diagnosed prenatally.
Although intralobar sequestrations are more common than extralobar
sequestrations, they typically present in adolescence with recurrent pneumonias.
In contrast, the extralobar location is less common but typically presents in the
fetal or neonatal period. An extralobar sequestration is located outside the lung
and has its own visceral pleura. Although it does not connect with the bronchial
tree, it may connect to the gastrointestinal tract. Approximately half of affected
infants have other anomalies, such as congenital diaphragmatic hernia, a
vertebral anomaly, or structural heart disease. Most lesions regress in utero or
postnatally.
Reference: Oermann CM. Bronchopulmonary sequestration. UpToDate.com
(Subscription required). Last Accessed on May 24, 2014
Pulmonology Question 9
A 7-day old intubated infant born at 24 weeks’ gestation has the following
chest radiograph:

Of the following, the most likely physiologic finding in this infant is:
A. Decreased airway resistance
B. Decreased dead space
C. Decreased lung compliance
D. Equal ventilation to perfusion ratio
Pulmonology Answer 9
C. Decreased lung compliance
The chest radiograph of the infant in this vignette reveals an extensive
bilateral ground-glass appearance of the lung parenchyma. In addition, the
presence of bubbly lucencies in the left lung is worrisome for pulmonary
interstitial emphysema (PIE). PIE is caused by alveolar rupture into the
peribronchiolar spaces. PIE can be diffuse or localized, often with an increased
appearance at the hilum. Infants with PIE have decreased lung compliance,
impaired gas exchange with a ventilation/perfusion mismatch, and increased
dead space.
Reference: Kirpalani H, Epelman M, Mernah JR. Imaging of the Newborn.
nd
2 edition. Cambridge University Press, 2012
Pulmonology Question 10
A 1-hour infant is admitted to the NICU with a mild degree of cyanosis and
mild respiratory distress. The infant is placed in an oxygen hood and requires
30% FiO2 to maintain an oxygen saturation over 94%. An AP chest radiograph
is obtained:

Of the following, the history that is LEAST likely to be found in this infant is:
A. Delivery by Cesarean section
B. Perinatal depression
C. Precipitous delivery
D. Prematurity
Pulmonology Answer 10
D. Prematurity
The chest radiograph of this infant is most consistent with retained fetal lung
fluid. Risk factors for transient tachypnea of the newborn include the following:
delivery by Cesarean section, perinatal depression, maternal sedation, and
precipitous delivery.
Pulmonology Question 11
An asymptomatic full-term infant is being evaluated for a murmur prior to
discharge to home. The infant’s chest radiograph is shown below:

Of the following, the most likely diagnosis in this infant is:


A. Medial pneumothorax
B. Pneumomediastinum
C. Pneumopericardium
D. Structural congenital heart disease
Pulmonology Answer 11
B. Pneumomediastinum
The chest radiograph of the infant in this vignette reveals an elevation of the
infant’s thymus as a result of mediastinal air, often called a “sail sign”. This
finding is not observed in infants with a medial pneumothorax or
pneumopericardium. A diagnosis of structural congenital heart disease cannot
be made based on this infant’s clinical and radiographic findings.
Pulmonology Question 12
A male infant born at 36 weeks’ gestation by vaginal delivery has severe
respiratory distress immediately after birth. Despite endotracheal surfactant
administration and maximal ventilator support, the infant remains hypoxemic
with a severe respiratory acidosis. An AP chest radiograph at 30 minutes of age
is obtained:

His echocardiogram at 1-hour of age reveals that the baby has a structurally
normal heart and a small patent ductus arteriosus with left-to-right shunting.
Of the following, the infant’s diagnosis can be found in infants with a history of
the following, EXCEPT:
A. Maternal diabetes
B. Maternal history of rupture of membranes at 24 weeks’ gestation
C. Posterior urethral valves
D. Thanatophoric dwarfism
Pulmonology Answer 12
A. Maternal diabetes
The AP view of this infant’s chest shows diffuse opacification of the infant’s
lung (left lung being more opacified compared with right lung) with a small
thorax, which is consistent with pulmonary hypoplasia. The radiographic
diagnosis of pulmonary hypoplasia correlates with this infant’s clinical findings
of severe respiratory acidosis and hypoxemia in the setting of a structurally
normal heart without pulmonary hypertension. Pulmonary hypoplasia can be
found in infants with:
Prolonged severe oligohydramnios from genitourinary anomalies or
prolonged rupture of membranes in mid-gestation
Intrathoracic masses such as congenital diaphragmatic hernia, congenital
pulmonary airway malformation, bronchogenic cyst, pulmonary
sequestration
Skeletal dysplasias, such as thanatophoric dwarfism
Maternal diabetes is not associated with pulmonary hypoplasia.
Pulmonology Question 13
A term infant with shoulder dystocia is admitted to the NICU because of
respiratory distress. The infant’s chest radiograph is shown below:

A repeat film is obtained 24 hours later and the findings are unchanged.
Of the following, the most appropriate evaluative study in this infant is:
A. Chest computed tomography
B. Chest magnetic resonance imaging
C. Chest ultrasonography
D. Lateral chest radiograph
Pulmonology Answer 13
D. Lateral chest radiograph
The chest radiograph of the infant in this vignette shows elevation of the right
hemidiaphragm and increased right perihilar markings. Right diaphragmatic
paralysis should be considered if the right hemidiaphragm is more than 2
intercostal spaces higher than the left diaphragm. Left diaphragmatic paralysis
should be considered if the left hemidiaphragm is more than 1 intercostal space
higher than the right diaphragm. Diaphragmatic paralysis occurs in infants with
a birth injury associated with lateral stretching of the neck. This diagnosis can
be confirmed by ultrasonography to assess for diaphragmatic movement.
Reference: Yerramilli Murty VSS, Ram KD. Phrenic nerve palsy: A rare
cause of respiratory distress in newborn. J Pediatr Neurosci. 2012;7:225-227
Pulmonology Question 14
A 4-day old female infant born at 24 weeks’ gestation is receiving
synchronized intermittent mechanical ventilation with a PIP of 19 cm H20, PEEP
of 5 cm H20, rate of 25 breaths per minute, and a FiO2 of 0.65. The neonatology
team obtains an AP chest radiograph shown below:

Of the following, the MOST preferred temporary position for this infant is:
A. Left side down
B. Prone
C. Right side down
D. Supine
Pulmonology Answer 14
C. Right side down
The chest radiograph of the infant in this vignette reveals diffuse hazy
granular opacities throughout the right lung and near complete opacification of
the left lung. The endotracheal tube tip is near the carina and the umbilical
venous catheter and umbilical arterial catheter are at T5. The neonatology team
can place the infant right side down for a short period of time to attempt to re-
expand the left lung. In addition, the team needs to pull back the endotracheal
tube and umbilical catheters so that they are in the appropriate positions.
Pulmonology Question 15
A female infant born at 30 weeks’ gestation has a prenatal diagnosis of a lung
mass, which is thought to be most consistent with a congenital pulmonary
airway malformation (CPAM) based on fetal MRI imaging. The neonatology
team assesses the infant after birth and notes that the infant has decreased breath
sounds over the right chest and severe respiratory distress with cyanosis
prompting bag-mask ventilation. After 5 minutes of positive-pressure
ventilation, the infant is intubated because of persistent cyanosis. The nurse
places an orogastric tube to remove gastric air because of abdominal distention
that occurred post-bagging. The infant is then brought to the NICU and a chest
radiograph is obtained:

Which of following statements about the infant in this vignette is true?


A. As all CPAMs contain adenomatoid and cystic tissue, these findings are
expected in this infant.
B. The CPAM is unlikely to connect with the tracheobronchial tree.
C. The infant requires close observation for several weeks awaiting
spontaneous resolution of the CPAM.
D. The CPAM most likely developed by the 7th week of gestation.
Pulmonology Answer 15
D. The CPAM most likely developed by the 7th week of gestation.
The chest radiograph of this infant demonstrates a right lung mass that was
diagnosed prenatally as a congenital pulmonary airway malformation (CPAM).
This malformation was previously termed congenital cystic adenomatoid
malformation but because only 1 of the 5 types is adenomatoid (type 3) and 3 are
cystic (types 1, 2 and 4), the term has recently been changed. CPAM is a
developmental lung abnormality that typically develops between the 5th and 7th
week of gestation; the heterogenous lesion is characterized by overgrowth of
bronchioles that communicates with an abnormal bronchial tree. Recent
histologic findings suggest that CPAM may result from an intrauterine airway
obstruction.
The diagnosis of CPAM is typically made in utero and close fetal
ultrasonographic monitoring is required to determine the size, content, and
distribution of the lesion. Fetal management with needle aspiration, shunt
placement, and/or resection may be required if the fetus develops hydrops.
Fortunately, most of these lesions regress by late pregnancy. Postnatal
management of symptomatic patients involves complete surgical resection.
Surgery for asymptomatic children remains controversial with advocates of
surgical management hoping to avoid potential complications such as recurrent
infections and development of focal dysplasias.
Reference: Lee EY, Dorkin H, Vargas SO. Congenital pulmonary
malformations in pediatric patients. Radiol Clin N Am. 2011;49:921-948 and
Tastekin E, Kaynar A, Ozdemir C, et al. Congenital pulmonary airway
malformation type 2: A case report with review of the literature. Turk Patoloji
Derg. 2014
Pulmonology Question 16
A neonatology team is present at the emergent Cesarean delivery of a term
infant with fetal bradycardia. There are no known sepsis risk factors. The infant
emerges through thin meconium and because the infant is apneic without any
spontaneous movements, the fellow intubates the infant to suction for
meconium. The fellow does not find any meconium below the vocal cords and
provides bag-mask ventilation. The infant requires 100% oxygen to maintain an
oxygen saturation of 95% and thus, the fellow re-intubates the infant and places
the infant on a significant amount of ventilator support. Upon arrival to the
NICU, the infant has a chest radiograph, shown below:

The neonatology fellow reviews the chest radiograph findings with the pediatric
resident.
Of the following, the most appropriate management of this infant includes:
A. Diuretic therapy
B. Inotropic support
C. Surfactant administration
D. Thoracentesis to drain the left pleural effusion
Pulmonology Answer 16
C. Surfactant administration
The chest radiograph of the infant in this vignette demonstrates coarse
reticular lung markings that can be found in infants with meconium aspiration
syndrome or pneumonia. The cardiac silhouette is not enlarged and there is no
pleural effusion or pneumothorax. The lung volumes are normal to increased.
In the setting of meconium passage and these radiographic findings, the infant
most likely has meconium aspiration syndrome. Because the aspirated
meconium can lead to a secondary surfactant deficiency, administration of
intubation is recommended. In addition, it is important to monitor for signs of
pulmonary hypertension.
Pulmonology Question 17
An infant born at 28 weeks’ gestation has severe respiratory distress. The
neonatology fellow intubates the infant and then places umbilical venous and
arterial lines in the infant. The infant’s chest radiograph is shown below:

The radiographic findings of the infant’s left lung are most likely attributable to:
A. Air within the pulmonary vascular tree
B. Collapsed alveoli around the bronchiolar airways
C. Pulmonary interstitial emphysema
D. Subcutaneous emphysema
Pulmonology Answer 17
B. Collapsed alveoli around the bronchiolar airways
The chest radiograph of the infant in this vignette reveals the bronchiolar
airways of the left lung, known as “air bronchograms”. This radiographic
finding occurs as a result of fluid-filled or collapsed alveoli surrounding the
airways, thus allowing air within the bronchioles to be visualized. The
endotracheal tube in this infant is at the carina and needs to be retracted. There
is atelectasis of the left lung and right upper lobe. Granular opacities are present,
consistent with collapsed alveoli. The umbilical venous catheter tip is likely to
be within the right atrium and the umbilical arterial catheter tip is seen with its
tip at thoracic vertebrae 6.
II. CARDIOLOGY
Cardiology Question 1
A term infant is noted to have severe cyanosis that does not respond to
oxygen supplementation. A loud end-systolic murmur is appreciated in the right
upper sternal border. The infant’s radiograph is shown below:

Of the following, the most likely cause of this infant’s murmur is:
A. Aortic valve insufficiency
B. Mitral valve stenosis
C. Pulmonary valve stenosis
D. Tricuspid valve stenosis
Cardiology Answer 1
C. Pulmonary valve stenosis
The chest radiograph of the infant in this vignette is slightly rotated but
reveals that the infant’s cardiac apex is pointing into the right chest rather than
the left. The infant’s gastric bubble is in the correct position in the left upper
quadrant of the abdomen. Thus, the infant has dextrocardia with normal
abdominal situs. The cardiothymic silhouette is slightly “boot-shaped”,
suggesting a possible diagnosis of tetralogy of Fallot. The infant’s murmur is
most likely to be pulmonary valvar stenosis because of the clinical finding of
cyanosis, timing during systole, location in an infant with dextrocardia, and
possible tetralogy of Fallot.
The remaining murmurs are appreciated during diastole. Below is a diagram
that shows the possible cardiac causes of murmurs (PDA-patent ductus
arteriosus; PS-pulmonary valvar stenosis; ASD-atrial septal defect; AS-aortic
valvar stenosis; VSD-ventricular septal defect; TR-tricuspid valve regurgitation;
MR-mitral valve regurgitation; IHSS-idiopathic hypertrophic subaortic stenosis)
at 4 locations (ULSB-upper left sternal border; URSB-upper right sternal border;
LLSB-left lower sternal border; and the apex).
Cardiology Question 2
A 15-day old female infant born at 27 weeks’ gestation has a chest radiograph
shown below (on the left) and another chest radiograph 15 hours later, shown
below (on the right):

Of the following, the most likely explanation for these cardiac and pulmonary
differences is:
A. Initiation of diuretics
B. Initiation of inhaled nitric oxide
C. Intubation
D. Ligation of the patent ductus arteriosus
Cardiology Answer 2
D. Ligation of the patent ductus arteriosus
The first chest radiograph shows diffuse opacities in both lungs, likely
representing a combination of atelectasis and edema. The heart size is difficult
to assess because of the rotation of the film but likely enlarged. The second film
reveals an endotracheal tube with the tip located a few millimeters above the
thoracic inlet. A new clip is seen in the expected position of the ductus
arteriosus and the heart is mildly enlarged. There has been interval clearing of
the pulmonary findings. There may be a tiny small left-sided pneumothorax.
Ligation of this infant’s patent ductus arteriosus is the most likely explanation of
these radiographic changes.
Cardiology Question 3
The neonatology team is called to the delivery room because of cyanosis and
respiratory distress in a full-term infant who is 10 minutes old. The infant’s
oxygen saturation measured in the right wrist is 55%. After intubation and
placement on 100% oxygen, the infant’s degree of cyanosis is unchanged. The
obstetrician reports that there are no concerns for chorioamnionitis, the amniotic
fluid was clear with membranes ruptured at delivery, and there were no birth
complications. The neonatologist orders a chest radiograph hoping to identify a
cause for the infant’s cyanosis:

Of the following, the most likely diagnosis in this infant is:


A. Meconium aspiration syndrome
B. Pulmonary hypertension
C. Total anomalous pulmonary venous return
D. Transient tachypnea of the newborn
Cardiology Answer 3
C. Total anomalous pulmonary venous return
This infant’s chest radiograph shows bilateral coarse reticular opacities, and
increased lung volumes. These findings are most consistent with either
meconium aspiration or pneumonia. However, since the infant’s clinical history
does not support these possibilities, the findings may also be attributable to an
underlying cardiac disease, such as total anomalous pulmonary venous return
(TAPVR). The findings of a small heart size and pulmonary venous congestion
observed on this chest radiograph suggest that this might be an obstructive type
of TAPVR (usually subdiaphragmatic) with decreased venous return but this will
need to be confirmed by echocardiography. Similar to the infant in this vignette,
infants with obstructive TAPVR present with respiratory distress and severe
cyanosis.
Cardiology Question 4
A full-term cyanotic male infant with hypotension and a concern for sepsis is
intubated because of respiratory distress. On examination, the infant has a
systolic murmur that is loudest over the upper left sterna border. The infant’s
chest radiograph is shown below:

Of the following, the most likely diagnosis in this infant is:


A. Tetralogy of Fallot
B. Total anomalous pulmonary venous return
C. Transposition of the great arteries
D. Truncus arteriosus
Cardiology Answer 4
A. Tetralogy of Fallot
The chest radiograph in the infant in this vignette reveals a “boot-shaped”
heart. This is found in patients with tetralogy of Fallot because of the upturned
cardiac apex as a result of right ventricular hypertrophy. Infants with tetralogy
of Fallot complicated by severe pulmonary stenosis have decreased pulmonary
vascularity, as evident on this infant’s film. The other answer options (i.e., total
anomalous pulmonary venous return, transposition of the great arteries, and
truncus arteriosus) are associated with cyanosis but affected infants should also
have increased pulmonary vascularity evident by chest radiographs.
Cardiology Question 5
The NICU team is called to the delivery of a male term infant with a prenatal
diagnosis of bilateral pleural effusions. The neonatology fellow observes that
the infant has severe respiratory distress and central cyanosis. She intubates the
baby and then performs a thoracentesis on the right side. After removing 50 mL
of fluid, the infant’s respiratory distress improves. Upon closer examination, the
fellow notes that the infant has a webbed neck, low-set ears, and a systolic
murmur loudest at the left upper sternal border.
Over the next several weeks, the infant requires multiple bilateral
thoracenteses. By 1 month of age he is requiring CPAP but tolerating full breast
milk feedings without reaccumulation of the pleural effusions. His CXR is
shown below:

Of the following, the most likely cardiac finding in this infant is:
A. Dysplastic pulmonary valve
B. Hypoplastic left ventricle
C. Supravalvar subaortic stenosis
D. Total anomalous pulmonary venous return
Cardiology Answer 5
A. Dysplastic pulmonary valve
Chylous effusions are often found in infants with trisomy 21 and Noonan
syndrome. Although the infant in this vignette had resolution of the chylous
effusions, the CXR showed persistent abnormal findings. These findings are
consistent with diffuse granular densities that may represent dilated lymphatic
vessels found in congenital lymphangiectasia. Congenital lymphangiectasia
results from abnormal development of pulmonary lymphatic vessels or
pulmonary lymphatic obstruction and may present with chylous effusions.
The findings of webbed neck and low-set ears suggest that the infant in this
vignette may have trisomy 21 or Noonan syndrome. Infants with trisomy 21 are
at increased risk of having structural heart defects, including complete
atrioventricular canal, septal defects, and tetralogy of Fallot while infants with
Noonan syndrome are at greater risk of having a dysplastic or thickened
pulmonary valve, cardiomyopathy, septal defects, or branch stenosis of
pulmonary arteries. A hypoplastic left ventricle, supravalvar subaortic stenosis,
and total anomalous pulmonary venous return are not typically associated with
trisomy 21 or Noonan syndrome.
Cardiology Question 6
A nurse is caring for a 2-day old female infant born at 24 weeks’ gestation
with surfactant deficiency and a history of a large left pneumothorax that
required thoracentesis and chest tube placement. Earlier that day, the chest tube
seemed to be working only intermittently and the neonatology fellow had placed
a 2nd left-sided chest tube. The nurse responds to the infant’s alarm that is
triggered as a result of bradycardia and hypoxemia. The infant appears pale,
cyanotic, and has poor perfusion. The nurse calls the neonatologist, neonatology
fellow, and respiratory therapist emergently to the bedside. The fellow is unable
to appreciate breath sounds on either side and the infant’s heart sides are
muffled. The fellow re-intubates the infant but the baby remains bradycardic
and hypoxemic, prompting initiation of chest compressions and administration
of intravenous epinephrine. An emergent chest radiograph is obtained, which is
shown below:

Of the following, the NEXT procedure that the fellow most likely performs is
(a):
A. Pericardiocentesis
B. Removal of air from the systemic circulation
C. Repositioning of the umbilical lines
D. Thoracentesis
Cardiology Answer 6
A. Pericardiocentesis
The chest radiograph of the infant in this vignette shows a residual left
pneumothorax that appears to be loculated in the subpulmonic location. There is
also an abnormal lucency of the right mid lung as well as a circumferential
lucency around the heart. These findings are concerning for a right-sided
pneumothorax and a pneumopericardium with a possible pneumomediastinum.
There is also air in the subcutaneous tissues of the left chest wall. The tip of the
endotracheal tub is just below the thoracic inlet and 2 left chest tubes have tips at
the medial aspect of the upper lung. The umbilical venous catheter has a tip at
the mid right atrium and the umbilical arterial catheter tip is at T6-T7.
Affected infants with pericardial air that is under tension will present with
acute circulatory collapse evident by severe cyanosis and hypotension. The air
in the pericardial sac will cause the heart sounds to be muffled. Pericardial
needle aspiration is required to improve this infant’s clinical status.
Cardiology Question 7
A term infant with respiratory distress requires intubation soon after birth.
Despite intubation, the infant remains cyanotic. The infant’s gas drawn from the
umbilical venous line is: pH=7.25, pCO2=60 mm Hg, pO2=250 mm Hg. The
infant’s chest radiograph is shown below:

Of the following, the most likely congenital heart defect in this infant is:
A. Infradiaphragmatic total anomalous pulmonary venous return
B. Tetralogy of Fallot
C. Transposition of the great arteries
D. Truncus arteriosus
Cardiology Answer 7
A. Infradiaphragmatic total anomalous pulmonary venous return
The film of the infant in this vignette shows an umbilical venous line
terminating at the inferior cavoatrial junction. The lungs have some streaky
diffuse bilateral opacities and the cardiomediastinal silhouette appears normal.
An infant with a high pO2 from a blood gas drawn from an appropriately
positioned high umbilical venous line may have infracardiac total anomalous
pulmonary venous return (TAPVR). Infants with infracardiac TAPVR have
pulmonary veins that drain into the venous system below the diaphragm (e.g.,
into the portal vein, hepatic vein or inferior vena cava). This type of TAPVR is
almost always obstructive. Because the well oxygenated pulmonary venous
blood enters the venous system, blood taken from a high umbilical venous line
reveals an elevated pO2. It is also possible that an umbilical venous pO2 can be
extremely high if the umbilical venous line is advanced too far with the distal
portal crossing the foramen ovale with the tip in the left atrium.
Cardiology Question 8
A pediatrician hears a systolic murmur in a well appearing 3-day old male
infant born at 37 weeks’ gestation. The infant’s pre- and post-ductal room air
oxygen saturations are 98% and the infant’s EKG shows a QRS axis of 120
degrees. The infant’s chest radiograph is shown below:

The parents are extremely concerned about the possibility of a congenital heart
defect. The pediatrician speaks with the family about the infant’s evaluation.
Of the following, the finding that is most suggestive of a possible congenital
heart defect in this infant is the:
A. Cardiothoracic ratio
B. Large thymus
C. QRS axis
D. Right-sided aortic arch
Cardiology Answer 8
D. Right-sided aortic arch
This infant’s chest radiograph reveals a right-sided aortic arch, which is
associated with a higher risk of structural heart disease. The cardiothoracic ratio
on an infant’s chest radiograph is not typically helpful as this ratio varies with
different lung expansions and is impacted by the thymic shadow. A normal QRS
axis in a newborn ranges between 100 and 150 degrees because of right
ventricular hypertrophy in a newborn and thus, is normal in the infant described
in this vignette. A large thymus is not associated with congenital heart disease
and is not noted in the chest radiograph of the infant in this vignette. Infants
with DiGeorge syndrome may have a small thymus and are at higher risk of
aortic arch abnormalities.
Cardiology Question 9
A male infant born at term gestation with a prenatal diagnosis of a large
ventricular septal defect has respiratory distress. The infant’s pediatrician orders
a chest radiograph:

Of the following, the most likely age of this infant is:


A. 1 hour of age because the placenta is no longer helping to oxygenate the
infant
B. 48 hours because the infant’s ductus arteriosus has recently closed
C. 6 weeks of age because the infant’s pulmonary vascular resistance has
decreased significantly
D. 3 months of age because the infant’s growth velocity is increasing
Cardiology Answer 9
C. 6 weeks of age because the infant’s pulmonary vascular resistance has
decreased significantly
The chest radiograph of the infant in this vignette shows a large heart and
increased pulmonary vascular markings as found in congestive heart failure.
This occurs in infants with a large ventricular septal defect when the pulmonary
vascular resistance decreases and the majority of shunting across the ventricular
opening is from the left to right ventricle resulting in increased pulmonary blood
flow. Because an infant’s pulmonary vascular resistance is lowest between 6 and
12 weeks of age, this is the most common timing for infants with a large
ventricular septal defect to develop symptoms of congestive heart failure.
Cardiology Question 10
A neonatology fellow is reviewing an infant’s cardiac issues with a 3rd year
medical student. The male infant was born at term and is a few hours old. He
has been intubated because of respiratory distress and has an orogastric tube in
place because of slight abdominal distention following period of bag-mask
ventilation. A loud murmur is appreciated and the infant has peaked P waves on
his EKG. The fellow challenges the medical student to try to guess the infant’s
likely structural heart defect based on the following chest radiograph:

Of the following, the cardiac valve that is most likely impacted in this infant is:
A. Aortic valve
B. Mitral valve
C. Pulmonary valve
D. Tricuspid valve
Cardiology Answer 10
D. Tricuspid valve
Based on the chest radiograph alone, this infant most likely has Ebstein’s
anomaly because of the enormous size of the heart and lack of pulmonary blood
flow. Infants with this anomaly have an abnormally displaced tricuspid valve
into the right ventricle with a right-to-left atrial shunt leading to decreased
pulmonary blood flow. As a result, the right atrium is quite large and the
cardiothoracic ratio on a chest radiograph is excessive with decreased pulmonary
blood flow. Affected infants may have a murmur from tricuspid insufficiency.
The infant’s EKG finding of peaked p waves corresponds to the right atrial
enlargement. Approximately 20% of infants with Ebstein’s anomaly have
Wolff-Parkinson-White on their EKG. Other causes of a large heart of this
radiographic size include cardiomyopathy (decreased forces on EKG) or arterio-
venous malformation; both of these diagnoses would be associated with
increased pulmonary vascular markings.
Cardiology Question 11
A term infant is evaluated in the NICU because of respiratory distress soon
after birth with decreased breath sounds over the right chest wall. The infant’s
initial chest radiograph is shown below:

The cardiology fellow performing the echocardiogram is unable to identify both


pulmonary arteries and consults with the cardiology attending who is able to
identify both pulmonary arteries. However, the attending observes that one
pulmonary artery is abnormal. A barium swallow study confirms the diagnosis.
Of the following, the most likely abnormality in this infant is that the:
A. Left pulmonary artery is arising from the right pulmonary artery
B. Obstruction of the left pulmonary artery
C. Obstruction of the right pulmonary artery
D. Right pulmonary artery is arising from the left pulmonary artery
Cardiology Answer 11
A. Left pulmonary artery is arising from the right pulmonary artery
A pulmonary artery sling is caused by an anomalous origin of the left
pulmonary artery. In this rare vascular anomaly, the left pulmonary artery
(designated “2” in Figure below) arises from the posterior portion of the right
pulmonary artery (designated “1” in Figure below) and courses between the
trachea and esophagus towards the left lung. Affected infants can present with
stridor, apnea, or cyanosis although some infants may remain asymptomatic.
The infant in this vignette likely has a pulmonary sling that compresses the right
mainstem bronchus leading to right-sided atelectasis.

Reference: Lee EY, Dorkin H, Vargas S. Congenital pulmonary


malformations in pediatric patients. Radiol Clin N Am. 2011;49:921-948
Cardiology Question 12
A term newborn has a sibling with a vascular abnormality. As a result, an
abdominal ultrasound is obtained with sagittal views of the inferior vena cava
and aorta (with and without Doppler imaging) shown below:

Of the following, the preferred management of this infant is:


A. Bisphosphonate therapy
B. Low molecular weight heparin therapy
C. Thrombolytic therapy
D. Vascular surgery
Cardiology Answer 12
A. Bisphosphonate therapy
The ultrasonographic images of this infant reveals diffuse calcification of the
wall of the aorta (labelled as “1” in Figure below) and a portion of a normal
inferior vena cava (labelled as “2” in the Figure below).

The Doppler view shows patency of the lumens of the aorta and inferior vena
cava. This aortic wall calcification is observed in infants with idiopathic
infantile arterial calcinosis, which is treated with bisphosphonate therapy.
Further ultrasonographic images of this infant’s abdomen would likely show
diffuse arterial calcification in the liver, spleen, and kidneys.
Cardiology Question 13
A neonatologist is performing a routine examination on a male infant born at
33 ½ weeks’ gestation who is now 2 days old. He hears an extremely loud
murmur. The infant is not cyanotic and does not have any respiratory symptoms
but because he is concerned about the quality of the murmur, he obtains a chest
radiograph, as shown:

The findings in this infant can be associated with all of the following EXCEPT
for:
A. Asplenia
B. Kartagener syndrome
C. Thymic hypoplasia
D. Polysplenia
Cardiology Answer 13
C. Thymic hypoplasia
The chest radiograph of the infant in this vignette reveals cardiomegaly with
dextrocardia. The lungs appear to be normally inflated with a mild hazy diffuse
opacity. The nasogastric tube terminates in the right-sided stomach and there is
an impression of a left-sided liver. If the abdominal organs are positioned as a
mirror image of the normal situs, then this infant has situs inversus. If the
abdominal organs are in an abnormal position that is not an exact mirror image
of the normal situs, the term situs ambiguous is used. It is difficult to tell situs
ambiguous from situs inversus based on a radiograph alone. Dextrocardia can
be associated with the following:
Asplenia, which is associated with bilateral “right-sidedness” that has 2
right lungs and is almost always associated with structural heart disease;
associated with a midline liver and right or left-sided stomach
Kartagener syndrome (i.e., immotile cilia syndrome), present in ~25% of
infants with situs inversus totalis (i.e., situs inversus and dextrocardia)
Polysplenia, which is associated with bilateral “left-sidedness” that has 2
left lungs, midline liver and right or left-sided stomach
Thymic hypoplasia is not a specific finding in patients with dextrocardia or
situs abnormalities; affected infants may have DiGeorge syndrome, which is
associated with congenital heart defects.
Reference: Jones KL, Crandall M, del Campo M. Smith’s Recognizable
Patters of Human Malformation. 7th edition. Saunders, 2013
Cardiology Question 14
A well appearing infant at 8 days of age has a loud systolic murmur
appreciated by a pediatrician during an outpatient pediatric visit. The
pediatrician orders a chest radiograph and an EKG, shown below:

Because of the quality of the murmur, the neonatologist then orders an


echocardiogram. This imaging study shows that the infant has an extremely large
muscular ventricular septal defect. The family meets with the cardiologist and
discusses the clinical symptoms that are likely to occur in this infant.
Of the following, clinical symptoms are most likely to develop in this infant
when:
A. The infant’s pulmonary vascular resistance is lower than the systemic
vascular resistance.
B. The infant’s Qp/Qs ratio is equal to or greater than 2.
C. Both A and B
D. Neither A nor B
Cardiology Answer 14
C. Both A and B
The chest radiograph of the infant in this vignette shows a normal heart size
and clear lung fields. The EKG shows a normal heart rate, normal rhythm,
appropriate right axis deviation for this infant’s age, normal T wave inversion in
leads V1 through V3 that is expected at 8 days of age and absence of left
ventricular hypertrophy. A normal EKG and chest radiograph are expected in an
infant with a large ventricular septal defect (VSD) at 1 week of age. The degree
of shunting across a VSD will depend on the infant’s pulmonary vascular
resistance and the size of the defect. As the pulmonary vascular resistance
decreases with time, the ventricular shunting will become left-to-right (left
ventricle to right ventricle) leading to increased pulmonary blood flow (with a Q
pulmonary blood flow/Q systemic blood flow ratio equal to or greater than 2)
and ultimately congestive heart failure. Symptomatic infants will demonstrate
an increased heart size and increased pulmonary vascular markings on the chest
radiograph and EKG findings of left ventricular hypertrophy as a result of left
ventricular volume overload. If the septal defect is large, with time, combined
ventricular hypertrophy may occur.
Cardiology Question 15
A growth-restricted term infant is noted to have a small pericardial effusion
and possible cardiomegaly prior to delivery. The infant initially appears well in
the delivery room but then at 5 minutes of age, becomes apneic and cyanotic and
requires bag-mask ventilation. The infant has some improvement in respiratory
effort and is placed on CPAP with 100% FiO2 prior to transport to the NICU.
The infant’s cardiac examination in the NICU is notable for a II/VI systolic
murmur, hyperactive precordium, and weak distal pulses. Because of severe
respiratory acidosis and persistent cyanosis, the infant is intubated and the
following chest radiograph is obtained:

The infant has severe hypotension that remains despite multiple colloid
boluses and inotropic agents. An emergent echocardiogram is performed that
shows severe ventricular dysfunction and hypertrophic cardiomyopathy
associated with a left ventricular outflow tract obstruction. Despite aggressive
management, the infant is unable to be resuscitated successfully and dies at 16
hours of age. The infant’s post-mortem cardiac findings are shown below:

A potential cause of this infant’s cardiomyopathy includes all of the following


EXCEPT:
A. Beckwith-Wiedemann syndrome
B. Fatty acid oxidation disorder
C. Pompe disease
D. Viral myocarditis
Cardiology Answer 15
D. Viral myocarditis
The infant in this vignette has clinical, radiographic, echocardiographic, and
post-mortem findings that are consistent with hypertrophic cardiomyopathy.
Causes of hypertrophic cardiomyopathy include the following:
Genetic disorders such as Noonan syndrome, Beckwith-Wiedemann
syndrome, Trisomy 21, Costello syndrome, Eagle Barrett syndrome, and
Friedreich’s ataxia
Metabolic disorders such as Pompe disease, fatty acid oxidation disorders,
and mitochondrial defects
A dilated cardiomyopathy in an infant is usually attributable to the following:
Myocarditis, typically viral
Familial dilated cardiomyopathy
X-linked dilated cardiomyopathy
Mitochondrial disorders
Reference: Wallis G, Fricker FJ. Neonatal cardiomyopathy. NeoReviews.
2012;13:e711-e723
III. ELECTROCARDIOGRAMS
Electrocardiograms Question 1
A pregnant woman is admitted to the hospital at 33 weeks’ gestation. She had
been to her obstetrician earlier that day for a routine appointment. The
obstetrician performed a bedside ultrasound that showed fetal tachycardia,
polyhydramnios, and bilateral pleural effusions. Because of the concern for
hydrops, the infant is delivered by Cesarean section. The infant emerges with
apnea and cyanosis but responds to bag-mask ventilation. The baby is then
placed on CPAP and the team transports the infant to the NICU. The infant’s
vital signs are: HR=290, RR=55, oxygen saturation in RA CPAP = 99%, BP =
63/46 (mean=54) mm Hg. An EKG is obtained, which is shown below:

The rhythm does not change with several different vagal maneuvers and the
infant’s nurse then places an IV. The neonatologist contacts the cardiologist and
then discusses the management plan with the father. The infant’s rhythm does
change after administration of an intravenous medication, as shown on the EKG
below:
1a. The neonatologist updates the infant’s father who wants to know about what
possible intrauterine therapeutic options would have been available for a fetus
with this rhythm. Of the following, the most preferred approach to
intrauterine therapy of this fetal arrhythmia is:
A. Administration of a beta-agonist to the pregnant woman
B. Administration of digoxin to the pregnant woman
C. Administration of steroids to the pregnant woman to prevent
progression of fetal symptoms
D. Intrauterine fetal pacing
1b. Of the following, the most likely mode of action of the medication
administered to this infant is:
A. Decreasing the conduction velocity of the atrioventricular node
B. Calcium channel activation
C. Constriction of vascular smooth muscle cells
D. All of the above
Electrocardiograms Answer 1
1a.B. Administration of digoxin to the pregnant woman
Intrauterine treatment for sustained, uninterrupted fetal supraventricular
tachycardia is indicated. At present, the first-line approach is typically
administration of digoxin to the pregnant woman. If this is unsuccessful, other
anti-arrhythmic agents such as flecainide, sotalol, and amiodarone can be used.
Elective delivery needs to be considered if a fetus develops hydrops, even if the
gestation is premature.
For fetuses with complete heart block, intrauterine interventions have not
been very successful. Options include: maternal beta-agonists (benefit not
proven), maternal steroids (for potential use in early in gestation for second-
degree atrioventricular block), and in utero pacing (case reports, not successful
long-term). Close fetal monitoring with weekly fetal echocardiograms is
necessary. If fetal hydrops develops, early delivery needs to be considered.
References: Killen SAS, Fish FA. Fetal and neonatal arrhythmias.
NeoReviews. 2008; 9:e242-e252;Yildirim A, Tunaodlu FS, Karaadac AT.
Neonatal congenital heart block. Indian Peds.2013;50:484-8
1b. A. Decreasing the conduction velocity of the atrioventricular node
This infant has an arrhythmia that is most consistent with supraventricular
tachycardia. Because this infant has a stable blood pressure, conversion to a
normal sinus rhythm with intravenous adenosine is indicated if vagal maneuvers
are unsuccessful. Adenosine has several modes of action, including:
Decrease in the conduction velocity of the atrioventricular node
Vascular smooth muscle relaxation
Calcium channel inhibitor
If the infant had a low blood pressure, cardiac conversion would have been
preferred instead of adenosine administration.
Reference: Klabunde RE. Adenosine. Cardiovascular Pharmacology
Concepts. Available at:
http://www.cvpharmacology.com/antiarrhy/adenosine.htm
Electrocardiograms Question 2
A pediatrician hears a soft systolic murmur in a well appearing term infant at
3 days of age. The pediatrician is not concerned about the infant but elects to
perform a cardiac evaluation in the NICU prior to discharging the infant to
home. The pediatrician is surprised to hear that the neonatologist has identified
an abnormality on the infant’s EKG, which is shown below:

Of the following, the most likely diagnosis of the infant in this vignette is:
A. Complete heart block
B. Hyperkalemia
C. Hypocalcemia
D. Wolff-Parkinson-White syndrome
Electrocardiograms Answer 2
D. Wolff-Parkinson-White syndrome
The EKG of the infant in this vignette shows 3 abnormalities:
Initial slurring of the QRS, called a delta wave (drawing shown below)
because the ventricular myocardium is activated early

Short PR interval and


Prolonged QRS complex
This is most consistent with Wolff-Parkinson-White (WPW) syndrome. Several
delta waves are highlighted on the infant’s EKG below:

A close-up of one of the Delta waves in this infant is shown below:

This rhythm occurs because of an electrical pathway between the atrium and
ventricle that bypasses the atrioventricular node.
Infants with complete heart block (i.e., third degree heart block) have
complete atrioventricular dissociation evident by independent atrial and
ventricular rates. Hypocalcemia leads to a prolonged QT interval.
Hyperkalemia is associated with the following electrocardiographic findings:

Reference: Park MK. The Pediatric Cardiology Handbook. 3rd edition; St


Louis: Mosby; 2003
Electrocardiograms Question 3
An infant is being followed by a cardiologist because of a strong family
history of prolonged QT syndrome. An echocardiogram has shown that that the
infant has a structurally normal heart. The infant’s most recent EKG is as
follows:

Based on the EKG findings above, the most likely age of this infant is:
A. 2 hours old
B. 2 days old
C. 2 weeks old
D. 2 months old
Electrocardiograms Answer 3
D. 2 months old
Because the right side of the heart provides most of the systemic cardiac
output in utero, a newborn is born with right ventricular hypertrophy. After
birth, with a decrease in pulmonary vascular resistance, closure of the foramen
ovale and ductus arteriosus, and increased systemic vascular resistance, the left
ventricle provides the systemic cardiac output and the left ventricle becomes
hypertrophied. This change in ventricular roles is evident by determining the
QRS axis over time of an infant without structural heart disease. The QRS axis
in a newborn with a structurally normal heart is typically between 90 and 180
degrees. At approximately 1 to 2 months of age, this axis changes to between 0
and 90 degrees. Because the QRS axis of infant in this vignette is between 0 and
90 degrees, of the options provided, the infant is most likely to be 2 months old.
Another finding that can help to determine the age of a patient based on the
patient’s EKG is the T waves in leads V1 through V3. Typically T waves are
upright in these leads in a newborn until 7 days of age when they invert. If there
is persistence of upright T waves in leads V1 through V3 beyond 7 days of age,
right ventricular hypertrophy may be present.
Reference: Goodacre S, McLeod K. Paediatric electrocardiology. BMJ.
2002;324:1382-1385
Electrocardiograms Question 4
A 3-day old infant has the following EKG:

Of the following, the most likely abnormality on this infant’s EKG is:
A. Decreased forces
B. Left superior QRS axis
C. Premature ventricular contractions
D. Prolonged QTc
Electrocardiograms Answer 4
D. Prolonged QTc
The EKG of the infant in this vignette reveals a prolonged QTc. Because the
QT interval is dependent on heart rate, one must calculate the QT interval by
accounting for the infant’s heart rate, using the following formula:

The infant in this vignette has a QT of 0.44 seconds and an RR interval of 0.64
seconds; using the above formula, this infant’s QTc is 0.55. Typically a QTc
value greater than 0.45 seconds is prolonged. This may be a normal finding in
infants less than 3 days of age. If the QTc is prolonged in a newborn greater
than age 3 days, possible etiologies include: hypokalemia, hypocalcemia,
hypothermia, or congenital prolonged QT syndrome.
The infant has a right inferior QRS axis, which is normal for age because of
right ventricular hypertrophy. The infant’s EKG does not show decreased forces
and there are no premature atrial contractions.
Reference: Goodacre S, McLeod K. Paediatric electrocardiology. BMJ.
2002;324: 1382-1385
Electrocardiograms Question 5
A nurse in the Newborn Nursery term is concerned about a newborn with a
low resting heart rate. She contacts the neonatology fellow who evaluates the
infant. On examination, the fellow notes that the infant is well appearing with a
normal examination. The fellow obtains an EKG, which is shown below:

Of the following, the most likely maternal antibodies that are present in this
infant are:
A. Anti-acetylcholine receptor antibodies
B. Anti-platelet antibodies
C. Anti-Ro/SSA and anti-La/SSB antibodies
D. Thyroid-stimulating hormone receptor blocking antibodies
Electrocardiograms Answer 5
C. Anti-Ro/SSA and anti-La/SSB antibodies
The EKG of the infant in this vignette shows atrioventricular dissociation
with this infant’s atrial rate of approximately 150 beats per minute and a
ventricular rate of approximately 65 beats per minute.

This arrhythmia may be associated with maternal Lupus as a result of anti-


Ro/SSA and anti-La/SSB antibodies that cross the placenta into the fetus and
injure the fetal conduction tissue.
Anti-acetylcholine receptor antibodies are transferred to neonates of mothers
with myasthenia gravis. Affected neonates have neurological dysfunction
associated with sucking, swallowing, breathing, and opening of eyelids; there is
minimal impact on the neonatal heart. Maternal anti-platelet antibodies may
cross the placenta and lead to thrombocytopenia in a fetus or neonate but does
not impact the fetus’/neonate’s cardiac function or rhythm. Neonates can
acquire thyroid-stimulating hormone receptor blocking antibodies from mothers
who have Graves’ disease. Affected infants may have clinical findings of
transient congenital hypothyroidism, which may be associated with a slow
resting heart rate.
Electrocardiograms Question 6
A cardiologist is reviewing an EKG of a 4-day old infant with a pediatric
resident:

Of the following, this infant most likely has:


A.Hyperkalemia
B.Left atrial enlargement
C.Left ventricular hypertrophy
D.Right atrial enlargement
Electrocardiograms Answer 6
D. Right atrial enlargement
The EKG of the infant in this vignette demonstrates a normal sinus rhythm,
peaked p waves, a normal QRS wave, and QRS axis of approximately 85
degrees. Peaked p waves that are taller than 2.5 mm (best visualized in lead II,
V1 or V4R) are found in infants with right atrial enlargement. One structural
heart defect with right atrial enlargement is Ebstein’s anomaly. Left atrial
enlargement is demonstrated on an EKG by increased width of the p wave
(typically over 0.1 seconds) and may be seen in patients with severe mitral
stenosis or regurgitation. Hyperkalemia may lead to a flat or absent p wave.
Left ventricular hypertrophy is apparent with increased R waves in lead V6 and
increased S waves in lead V1, which are not seen in the EKG of this infant.
Reference: Goodacre S, McLeod K. Paediatric electrocardiology. BMJ.
2002;324:1382-1385
O’Connor M, McDaniel N, Brady MJ. The pediatric electrocardiogram. Am J
Emerg Med. 2008;26:221-228
Electrocardiograms Question 7
A neonatologist is in the process of transferring a 7-day old newborn with
cyanotic heart disease to the cardiac intensive care unit. He finds that the baby’s
EKG (shown below) correlates with the prenatal cardiac finding.

Of the following, the most likely cardiac diagnosis of the infant in this vignette
is:
A. Pulmonary atresia
B. Tetralogy of Fallot
C. Tricuspid atresia
D. Truncus arteriosus
Electrocardiograms Answer 7
C. Tricuspid atresia
The EKG of the infant in this vignette demonstrates a QRS axis of
approximately -90 degrees. A normal QRS axis is a newborn is between 90 and
180 degrees. This baby’s abnormal axis can be associated with tricuspid atresia
(with decreased right ventricular forces) or a complete atrioventricular canal. In
contrast to tricuspid atresia, newborns with pulmonary atresia and truncus
arteriosus will initially have a normal QRS axis. Infants with tetralogy of Fallot
have persistent right ventricular hypertrophy and a QRS axis typically between
90 and 180 degrees, independent of age.
Electrocardiograms Question 8
An infant with prolonged QT syndrome is being monitored closely with daily
EKGs. The neonatology fellow shows the following EKG to the attending
cardiologist who states that the infant needs to be urgently transferred to the
Cardiac Intensive Care Unit.

Of the following, the most likely EKG finding to explain the cardiologist’s
reaction is:
A. Abnormal U waves
B. Extremely prolonged QTc
C. Notched T waves
D. T wave alterans
Electrocardiograms Answer 8
D. T wave alterans
Features of prolonged QT syndrome include: notched T waves, abnormal U
waves, and T wave alterans. The EKG of the infant in this vignette shows T
wave alterans with alternating upright and inverted T waves throughout (see blue
arrows below). This finding places infants at risk of a ventricular arrhythmia
and sudden cardiac demise.

Reference: Goodacre S, McLeod K. Paediatric electrocardiology. BMJ.


2002;324:1382-1385
Electrocardiograms Question 9
A 6-week old infant born at 33 weeks’ gestation has radiographic findings of
congestive heart failure. The neonatology team performs an initial cardiac
evaluation (pre- and post-ductal oxygen saturations, 4 extremity blood pressures,
and an EKG) and then consults with the pediatric cardiology service. The
infant’s EKG is shown below:

Prior to reviewing the EKG with the cardiologist, the neonatologist reviews
the EKG with the pediatric resident. The resident identifies several q waves.
Of the following, the most likely cause of this infant’s q waves is:
A. ALCAPA (anomalous origin of the left coronary artery from the
pulmonary artery)
B. Lateral wall infarction
C. Non-specific finding
D. Right ventricular hypertrophy
Electrocardiograms Answer 9
C. Non-specific finding
Q waves can be present in leads II, III, aVF, V5 and V6 in newborns without
any significant associated problem. If q waves are present in other leads, this is
concerning for an underlying problem. For example, if deep and narrow q
waves are visible in leads I, aVL, V4, V5 and V6, this may be related to
ALCAPA. Q waves can also be found in patients with a myocardial infarction
with lead involvement corresponding to the area of infarction. For example, q
waves in leads V1 through V6 correspond to an anterior wall infarction; q waves
in leads II, III and aVF with an inferior wall infarction; and q waves in leads I
and aVL are associated with a lateral wall infraction.
References:
Goldberger AL. Electrocardiogram in the diagnosis of myocardial ischemia
and infarction. UpToDate (Subscription required). 2014;
Goodacre S, McLeod K. Paediatric electrocardiology. BMJ. 2002;324:1382-
1385;
Varghese MJ, Kothari SS. The caveats in the diagnosis of anomalous origin
of left coronary artery from pulmonary artery. Paediatr Cardiol. 2010;12:3-8
Electrocardiograms Question 10
A term infant is admitted to the Special Care Nursery for a sepsis evaluation.
The baby’s nurse observes that the infant has an irregular heart rate on
examination that correlates with the monitor reading. The neonatologist reviews
the infant’s EKG:

Of the following, the most preferred approach to this infant’s finding is:
A. Daily EKGs until normalization
B. Emergent consultation with the pediatric cardiologist
C. Further intervention is not required
D. Obtain a family history of arrhythmias
Electrocardiograms Answer 10
C. Further intervention is not required
The EKG of the infant in this vignette reveals premature atrial contractions
(PACs). PACs are a common finding in newborns and are typically benign.
Most PACs are associated with a normal QRS morphology, as shown in this
infant’s EKG.
One of the PACs in this infant (shown in the green box) occurs while the
atrioventricular (AV) node is partially repolarized, resulting in a different QRS
wave morphology. If a PAC occurs when the AV node is refractory, the impulse
will be blocked and will not be conducted to the ventricle, leading to a non-
conducted PAC (shown in the blue box). In this case, the next QRS wave is still
in step with the regular ventricular rate.

Reference: Cloherty JP, Eichenwald EC, Hansen AR, Stark AR. Manual of
Neonatal Care. Lippincott Manual Series. 2011
Electrocardiograms Question 11
A female infant is born at 32 weeks’ gestation and admitted to the NICU. She
is placed on CPAP because of respiratory distress and is started on peripheral
intravenous total parenteral nutrition. The infant’s nurse observes several
abnormal beats on the cardiovascular monitor and reports these findings to the
neonatology fellow. The neonatology fellow orders an EKG, which is shown
below:

Of the following, the most preferred approach to this infant’s EKG findings is:
A. Daily EKGs until normalization
B. Emergent consultation with the pediatric cardiologist
C. Further intervention is not required
D. Obtain a family history of arrhythmias
Electrocardiograms Answer 11
C. Further intervention is not required
Premature ventricular contractions (PVCs) may occur randomly or at specific
intervals. The EKG of this infant demonstrates PVCs that occur every 3rd beat;
this rhythm is termed “trigeminy”.

If PVCs occur every 2nd beat, this is termed “bigeminy”. If two PVCs occur
sequentially, this is known as a “couplet”. These rhythms are not concerning
unless an infant has clinical symptoms in which case, suppression with
medications may be required. Ventricular tachycardia is defined as 3 or more
PVCs in a row and requires immediate evaluation and management.
Electrocardiograms Question 12
A full-term female infant with a prenatal diagnosis of trisomy 21 and a
complete atrioventricular canal is admitted to the NICU because of respiratory
distress, cyanosis, and poor perfusion. She requires intubation, multiple normal
saline boluses, inotropic support, red blood cell transfusions, and platelet
transfusions. The neonatology team has a broad differential for the cause of this
infant’s illness. An arrhythmia is noted on the cardiorespiratory monitor and an
EKG is obtained, shown below, which suggests decreased voltages.

Of the following, the diagnosis that is most supported by the EKG findings of
this infant is:
A. Coxsackie disease
B. Electrolyte imbalance
C. Metabolic disorder
D. Pseudomonas sepsis
Electrocardiograms Answer 12
A. Coxsackie disease
Although the EKG of the infant in this vignette does not reveal an arrhythmia,
it does demonstrate decreased voltages. This can be found in infants with:
myocarditis, cardiomyopathy, or pericardial effusion. Coxsackie disease is the
most common cause of myocarditis in the newborn and can lead to clinical
findings of respiratory distress, hypotension, thrombocytopenia, and anemia, as
found in the infant in this vignette.
While electrolyte imbalances can lead to abnormal EKG intervals, this
infant’s EKG does not reveal any prolonged or shortened intervals. Metabolic
disorders are not associated with any specific EKG findings. Although
Pseudomonas sepsis can lead to the clinical findings observed in this infant, this
illness does not have any specific EKG findings.
Electrocardiograms Question 13
A pediatrician notices that a 1-week old neonate has a low resting heart rate
during a routine examination. She reviews the infant’s EKG, which is shown
below:

Of the following, the pediatrician’s most likely assessment of this infant’s EKG
is:
A. Complete heart block
B. Mobitz type I
C. Mobitz type II
D. Normal sinus rhythm with a low resting heart rate
Electrocardiograms Answer 13
D. Normal sinus rhythm with a low resting heart rate
This infant’s EKG reveals a normal sinus rhythm with a QRS wave following
every p wave. The heart rate is slightly slow for a newborn, estimated at
approximately 100 beats per minute. This rate can be estimated by using the
beats per minute pattern of 300, 150, 100, 75, 60 and 50 per each dark red
square. Alternatively, the heart rate can be calculated by dividing 60 by the RR
interval measured in seconds. Thus, for the EKG of the infant in this vignette,
the RR distance is 15 small boxes x 0.04 seconds per box = 0.6 seconds. Thus,
60 divided by 0.6 yields a heart rate of 100 beats per minute. The PR interval of
the infant is this vignette is within normal limits.
There are several types of atrioventricular (AV) conduction abnormalities.
First degree AV block is associated with a prolonged PR interval because of AV
nodal delay. In second degree AV block, the atrial impulses are not conducted
consistently to the ventricle. There are 2 types of second degree block: Mobitz
type I and Mobitz type II. Mobitz type I (also known as Wenckebach rhythm)
has increasing PR intervals until an atrial impulse is not conducted, leading to an
absent QRS complex. In Mobitz type II, although the PR intervals are
consistent, a random p wave is not conducted because there is random failure of
AV conduction. Complete AV block (also known as third degree AV block) is
caused by complete AV dissociation with independent ventricular and atrial
rates. The Table below shows electrocardiographic findings of these abnormal
rhythms.

Reference: Park MK. The Pediatric Cardiology Handbook. 3rd edition; St


Louis: Mosby; 2003
Electrocardiograms Question 14
A term infant at 8 days of age has an EKG with some findings that are
consistent with biventricular hypertrophy.

Of the following, the finding on this EKG that is most consistent with right
ventricular hypertrophy is:
A. Inverted T waves in lead V1
B. Large R waves in lead V1
C. Large R waves in lead V6
D. Positive T waves in lead V6
Electrocardiograms Answer 14
B. Large R waves in lead V1
Right ventricular hypertrophy is evident on a newborn EKG when:
The R waves in lead V1 are greater than the 98th percentile for age (>25.5
mm for newborn age 0 to 7 days, > 20.8 mm for infants 1 to 3 weeks of
age) and
The S waves in lead V6 are greater than the 98th percentile for age (>9.6
mm for newborn age 0 to 7 days, >9.8 mm for infants 1 to 3 weeks of age)
Left ventricular hypertrophy is evident on a newborn EKG when:
The R waves in lead V6 is greater than the 98th percentile for age (greater
than 11.8 mm for newborn age 0 to 7 days, >16.4 mm for infants 1 to 3
weeks of age) and
The S waves in lead V1 are greater than the 98th percentile for age (greater
than 9.6 mm for newborn age 0 to 7 days, > 9.8 mm for infants 1 to 3
weeks of age).
The EKG of the infant in this vignette has definite evidence of right ventricular
hypertrophy and criteria are almost consistent with left ventricular hypertrophy
as well.
Typically T waves are upright in leads V1, V2 and V3 in a newborn until 7
days of age when they invert. If there is persistence of upright T waves in leads
V1 through V3 beyond 7 days of age, this is consistent with right ventricular
hypertrophy. If T waves are deflected negatively in leads V5 and V6, this is
consistent with left ventricular hypertrophy. Neither of these abnormalities is
evident on this infant’s EKG.
References: Goodacre S, McLeod K. Paediatric electrocardiology. BMJ.
2002;324:1382-1385;
Galli MA, Danzi GB. A Guide to Neonatal and Pediatric ECGs. Springer,
2010;
Tschudy MM, Arcara KM. The Harriet Lane Handbook. 19th edition. Elsevier.
2012
Electrocardiograms Question 15
A term infant is born to a woman with concerns for chorioamnionitis. The
infant is brought to the NICU for a sepsis evaluation. The infant has an elevated
temperature and tachycardia. After the infant has been monitored for a few
minutes, the nurse responds to an alarm based on the findings on the
cardiorespiratory monitor. The rhythm strip of this infant’s EKG is shown
below:

Of the following, the most preferred approach to this infant’s finding is:
A. Daily EKGs until normalization
B. Emergent consultation with the pediatric cardiologist
C. Further intervention is not required
D. Obtain a family history of arrhythmias
Electrocardiograms Answer 15
B. Emergent consultation with the pediatric cardiologist
The infant in this vignette has tachycardia with a period of broad QRS
complexes that can be caused by one of the following:
Ventricular tachycardia
Supraventricular tachycardia with a bundle branch block
Atrioventricular re-entry tachycardia
Emergent consultation with the pediatric cardiologist is necessary to assist with
treatment. If the infant is stable, the cardiologist will determine whether
adenosine or an anti-arrhythmic agent should be given. If the infant is unstable,
emergent DC cardioversion is needed.
Neonates with ventricular tachycardia may present with congestive heart
failure. Associated etiologies include structural heart disease, electrolyte
abnormalities, myocarditis, cardiac tumors, prolonged QT syndrome, and
cardiomyopathy. The most common cause in neonates is idiopathic.
Reference: Kothari DS, Skinner JR. Neonatal tachycardias: An update. Arch
Dis Child Fetal Neonatal Ed. 2006;9:F136-144
IV. NEUROLOGY
Neurology Question 1
A full-term infant was born by vacuum-assisted vaginal delivery. During the
initial newborn assessment, the infant’s nurse noted swelling of the head. She
asked the Pediatric Resident to evaluate. The Resident was unsure about the
extent of the swelling and ordered a head MRI.

Of the following, the preferred NEXT step in the management of this infant is:
A. Admit the infant to the intensive care unit for frequent vital signs and
monitoring for shock
B. Contact neurosurgery for immediate evacuation of subdural blood
C. Discuss with the family that this is a common newborn finding, and
although it is expected to resolve by 1 month of age, calcification may
occur during the resolution phase due to subpericranial osteogenesis
D. Reassure the nurse that the findings are consistent with a caput
succedaneum and continue routine newborn care
Neurology Answer 1
A. Admit the infant to the intensive care unit for frequent vital signs and
monitoring for shock
The Figure below identifies the unique locations for the common
cranial/scalp lesions in a newborn that result during the delivery process.
Although the thin layers of the skin, aponeurosis, and periosteum are hard to
discern, the location of the fluid/blood collection in this vignette is not consistent
with either a caput (A) or cephalohematoma (B). Rather, the large collection
covering the parietal and temporal areas suggest a subgaleal hemorrhage (C),
which can lead to rapid blood loss, coagulopathy, and shock warranting close
observation in the intensive care unit. The fluid collection appears outside of the
skull and thus a subdural hemorrhage is unlikely. Statement D above refers to a
cephalohematoma.
Neurology Question 2
A routine 7-day head ultrasound (HUS) was performed in a 28-week
gestational age preterm infant
Of the following, the preferred NEXT step in the management of this infant is:
A. Discuss with the family the presence of a grade IV hemorrhage causing
posterior lateral ventricular dilation
B. Evaluate for other associated findings commonly seen with an absent
corpus collosum
C. Order an MRI to further clarify a possible cerebellar hemorrhage
D. Schedule a follow-up HUS at approximately 30 days of age
Neurology Answer 2
D. Schedule a follow-up HUS at approximately 30 days of age
The image for this case represents a normal sagittal view of a cranial
ultrasound without any signs of a hemorrhage.
Neurology Question 3
The following MRI is representative of a:

A. A communicating encephalomeningocele with hydrocephalus


B. A communicating meningocele with hydrocephalus
C. A non-communicating encephalomeningocele with hydrocephalus
D. A non-communicating meningocele with hydrocephalus
Neurology Answer 3
B. A communicating meningocele with hydrocephalus
Encephaloceles are neural tube defects of the brain. In North America, this
defect is more likely to occur in the back of the head. If the defect only contains
meninges and spinal fluid it is called a meningocele. If brain tissue is also
involved, the defect is called an encephalomeningocele. In this image the defect
has the same contrast as the spinal fluid and no brain tissue is apparent. Finally,
an open fistula or communicating tract extends from the posterior fossa
superiorly and through the dural reflections of the superior sagittal sinus.
Neurology Question 4
A full-term infant was born to a 31-year old G3P2 woman. The obstetric team
provides routine drying and stimulation in the delivery room. The infant’s Apgar
score is 9 (1minute) and 9 (5minutes). On physical exam, the infant’s head is
noted to be large and the cranial sutures are widely spaced. An MRI of the head
is obtained with one image shown below:

Of the following, the statement that is false for this clinical scenario is:
A. Cranial transillumination is pathognomonic for this condition
B. Possible etiologies include genetic (including syndromic), vascular
occlusion/abnormalities, and infectious
C. The condition is rare occurring approximately in 1 per 10,000 births
D. The majority of patients will die within 2 years of age
Neurology Answer 4
A. Cranial transillumination is pathognomonic for this condition
This infant has hydranencephaly and while positive transillumination is a
hallmark feature, it is not pathognomonic, a sign or symptom that is so
characteristic of a disease that it can be used to make a diagnosis. Other
conditions that may present in a similar manner, which should be evaluated
including severe hydrocephalus and holoprosencephaly.
Reference: Chinsky JM. Hydranencephaly: Transillumination may not
illuminate diagnosis. NeoReviews Vol.13 No.4 April 2012.
Neurology Question 5
A routine HUS at 1 week of age in a 27 week gestational age preterm infant is
obtained.

Of the following, the statement that is most correct for this clinical scenario is:
A. Anticipate an increase in the risk of long-term neurodevelopmental
impairment
B. Begin an evaluation for an associated aneuploidy
C. Discuss with the family the presence of an intraventricular hemorrhage
requiring follow-up within 1 week
D. Explain to the family that this is an incidental finding with an expectation
for complete resolution
Neurology Answer 5
D. Explain to the family that this is an incidental finding with an expectation for
complete resolution
This infant’s ultrasound reveals a choroid plexus cyst at the frontal tip/caudal-
thalamic grove of the choroid plexus. Though, when these anatomical findings
where first recognized it was unclear what their biological significance was,
multiple long-term follow-up studies have determined that these cysts are benign
without long-term consequence.
Neurology Question 6
A Neonatology Fellow is evaluating a former 29 week gestational age preterm
infant who is now 9 months’ corrected age. On neurological exam the infant’s
overall gross motor and cognitive development is delayed. The infant is
hypotonic and when reaching for a small bead demonstrates an immature pincer
grasp and an intention tremor. An MRI of the head is obtained to guide further
diagnosis and management.

The following are all appropriate next steps, EXCEPT:


A. Discuss with the family that the recurrence risk can be as high as 25%
B. Inform the family that the findings are not related to the infant’s premature
birth
C. Obtain a detailed family history and begin an evaluation for Werdnig-
Hoffmann disease, Tay-Sachs disease, and Menkes disease
D. Order a follow-up audiology assessment even though the infant passed a
hearing screen before discharge from the intensive care unit
Neurology Answer 6
B. Inform the family that the findings are not related to the infant’s premature
birth
The significant finding in this case is cerebellar hypoplasia. In the side-by-
side image, the infant with the small cerebellum is on the left and the infant with
a normal size cerebellum is on the right. The abnormal cerebellum is clearly
smaller in size filling less of the posterior fossa space.

Infants with cerebellar hypoplasia often manifest developmental delay, speech


delay, seizures, hypotonia, hypertonia, ataxia, impaired coordination, strabismus,
and nystagmus. Etiologies are varied and include genetic, ischemic, and
infectious factors. Premature infants are at risk for cerebellar hypoplasia
secondary to hemorrhage and/or hypoxia. Interestingly, a well-described, rare
autosomal recessive entity of cerebellar hypoplasia has been characterized.
Neurology Question 7
An infant girl is born at 38-5/7 weeks’ gestation by vacuum-assisted cesarean
section due to a poor biophysical profile. Her Apgar score is 7 at one minute and
8 at five minutes. The venous cord pH is 7.20 and the arterial cord pH is 7.08.
At 5 hours of age, the infant is transferred to the intensive care unit for
management of hypoglycemia and focal seizures evident by repetitive right arm
movement.

The following are all appropriate next steps, EXCEPT:


A. Administer a 20 mg/kg loading dose of phenobarbital
B. Discuss with the family that defining the time of injury is difficult and it is
possible that the event occurred in utero
C. Obtain an echocardiogram and renal Doppler studies
D. Offer formula to correct the hypoglycemia and recheck within 30 minutes
after the feeding, if the glucose concentration is below 40 mg/dL then
initiate intravenous dextrose fluids
Neurology Answer 7
D. Offer formula to correct the hypoglycemia and recheck within 30 minutes
after the feeding, if the glucose concentration is below 40 mg/dL then initiate
intravenous dextrose fluids
The major finding in this case is a large ischemic infarct in the left
hemisphere. The loss of cortical ribbon in the image below (red arrow) is
consistent with this diagnosis.

Seizure management as well as treatment of the hypoglycemia is of


paramount importance. In a baby with neurological symptoms, oral feeding trials
should not be used to correct the hypoglycemia. An embolic etiology of the
infarct must be considered and an echocardiogram and renal Doppler studies will
aid in that evaluation. Timing an ischemic cerebral injury is always difficult and
should be done with caution. The observation that the injury is so well-defined at
5 hours of life leaves open the possibility that this may have occurred in utero
versus being an acute injury.
Neurology Question 8
Label 5 anatomic structures.

(1) _____________________________
(2) _____________________________
(3) _____________________________
(4) _____________________________
(5) _____________________________
Neurology Answer 8
Neurology Question 9
A full-term infant presents to the intensive care unit with an increased head
circumference, widely split sutures, and poor feeding. A head MRI is obtained
with one image shown below:

Of the following, the statement that is false for this clinical scenario is:
A. In contrast to brain tumors diagnosed in later infancy and childhood, the
prognosis for long-term survival is favorable.
B. Neonatal brain tumors are uncommon and represent only 2% of all
pediatric brain tumors.
C. The most common postnatal, clinical presentation is an increased head
circumference for age.
D. The most common tumor type is a teratoma and the most common location
is supratentorial.
Neurology Answer 9
A. In contrast to brain tumors diagnosed in later infancy and childhood, the
prognosis for long-term survival is favorable.
Unfortunately, the head MRI of the infant in this vignette demonstrates a
mass that is most likely a brain tumor.

The overall prognosis of newborns with brain tumors is very poor. Fortunately,
the occurrence is rare and represents only 0.5-1.9% of all pediatric brain tumors.
Reference: Buetow PC, Smirniotopoulos JG, Done S. Congenital brain
tumors: a review of 45 cases. AJR Am J Roentgenol. 1990 Sep;155(3):587-93.
Neurology Question 10
A head MRI was obtained in a full-term infant born to a G1P0, 28-year old
woman for persistent thrombocytopenia and a failed hearing screen.

Of the following, the statement that is most INACCURATE for this clinical
scenario is:
A. Asymptomatic infants are at equal risk as symptomatic infants for hearing
loss
B. It is likely that the mother had some exposure to cats during her pregnancy
C. The maternal-to-fetal transmission is 25x higher for primary versus non-
primary transmission
D. The risk of fetal anomalies is greatest with infection in the first trimester
Neurology Answer 10
B. It is likely that the mother had some exposure to cats during her pregnancy
Present in this image are periventricular calcifications, which is a common
finding in an infant with a cytomegalovirus (CMV) infection. This T2 Gradient
Echo image shows multiple small black dots bilaterally in the periventricular
region that represent multiple small calcifications (red arrows shown in the
image below). Each of the statements above, except for A, is true of CMV.
Statement A is true of toxoplasmosis, which also has intracranial calcifications,
but not typically in the periventricular distribution.
Neurology Question 11
A head ultrasound is obtained in a full-term male infant with macrocephaly:

In counseling the family, the following statements are accurate except:


A. For a third of the patients, the hydrocephalus is obstructive and will
require shunt placement.
B. Long-term neurodevelopmental outcomes can be variable.
C. The recurrence risk is very low.
D. This is the most common form of congenital hydrocephalus.
Neurology Answer 11
C. The recurrence risk is low
This image shows severe congenital hydrocephalus and the most common
cause in a newborn is X-linked Aqueductal stenosis (stenosis of the aqueduct of
Sylvius. Non-genetic causes of aqueductal stenosis include viral infections
(mumps, rubella, parainfluenza) and an association with Arnold-Chiari
malformation. Long-term neurodevelopmental outcomes are contingent on the
presence of other abnormalities.
Neurology Question 12
The following cerebellar scan was obtained within the first several days in a
25 week gestational age infant.

The following statements are accurate of this finding, except:


A. If isolated, global neurocognitive impairment is unlikely.
B. Risk is inversely related to gestation age.
C. The majority of the cases are unilateral.
D. The posterolateral approach improves detection of these lesions.
Neurology Answer 12
A. If isolated, global neurocognitive impairment is unlikely.
This head ultrasound image is a posterolateral view of the cerebellum and
reveals a cerebellar hemorrhage (in the selected area on the top of the image).
Cerebellar hemorrhage occurs in approximately 10% of extremely low
gestational age newborns. Even if isolated, the presence of cerebellar
hemorrhage profoundly affects neurodevelopmental outcome in multiple
domains – cognition, motor, and behavior – a testament to the diverse, vital
functions of the cerebellum.
Reference: MM Zayek. Cerebellar hemorrhage: a major morbidity in
extremely preterm infants. Journal of Perinatology (2012) 32, 699–704.
Neurology Question 13
A routine HUS in a moderately preterm infant reveals the following image.

Associated findings may include the following, except:


A. Cerebral ischemia
B. Cranial bruit
C. Multiple birth
D. Thrombocytopenia
Neurology Answer 13
B. Cranial bruit
This is an image of lenticulostriate vasculopathy (LSV), calcification of deep
cerebral vessels. This is a rare finding detected in 0.4% of all neonates and
increased to 6% in critically ill neonates. Some proposed etiologic factors are
viral infections such as CMV, chromosomal abnormalities, hypoxemia-ischemia,
fetal drug exposure, twin-to-twin transfusion syndrome, and congenital heart
disease.
Of the choices above, thrombocytopenia can suggest CMV infection and
multiple births increase the potential for twin-to-twin transfusion syndrome.
Cerebral ischemia is a major risk factor. The presence of a cranial bruit would
suggest a Vein of Galen malformation and is not associated with LSV.
Reference: Makhoul IR et al. Neonatal lenticulostriate vasculopathy: further
characterization. Arch Dis Child Fetal Neonatal Ed 2003;88:F410–F414
Neurology Question 14
A head MRI is obtained in an infant with an abnormal head shape.

Associated historical and clinical findings in infants at risk for this skull
deformation may include the following, except:
A. Breech positioning
B. Oligohydramnios
C. Premature closure of the metopic suture
D. Prematurity
Neurology Answer 14
C. Premature closure of the metopic suture
This characteristic skull shape is termed dolichocephaly. Each of the factors
above, except C, is a risk factor for dolichocephaly. Premature closure of the
sagittal suture leads to this shape; whereas, premature closure of the metopic
suture leads to another skull deformation termed trigonocephaly. Of note, this
image also reveals mild hydrocephalus.

Reference: Printed with permission from: Brodsky D, Martin C. Neonatology


Review. 2nd edition. Lulu. 2010
Neurology Question 15
A routine 7-day head ultrasound (HUS) was performed in a 27-week
gestational age preterm infant.

Of the following, the preferred NEXT step in the management of this infant is:
A. Discuss with the family the presence of an intraventricular hemorrhage
may impact long-term neurodevelopment
B. Discuss with the team the role of serial spinal taps to minimize the risk for
posthemorrhagic hydrocephalus
C. Order an MRI to evaluate for extent of injury and other associated findings
D. Schedule a follow-up HUS at approximately 30 days of age
Neurology Answer 15
A. Discuss with the family the presence of an intraventricular hemorrhage may
impact long-term neurodevelopment
The image demonstrates a subtle grade I hemorrhage (arrow). Although this
is grade 1, the scientific literature suggests this may still impact long-term
outcomes. There is no need for an MRI at this time. The next HUS should be
within the next 1-2 weeks; waiting 1 month can miss evolution of the
hemorrhage. Finally, there are no signs of post-hemorrhagic hydrocephalus.
Neurology Question 16
The family asks to speak to the doctor about their infant’s odd skull shape. A
radiographic image of the infant’s skull is shown below:

The following are acceptable actions in this case, except:


A. Keep the infant in the Newborn Nursery for continued routine newborn
care
B. Prepare the family that resolution may be take several weeks to over a
month
C. Reassure the family that this never results in any long-term complications
of the cranium or brain
D. Warn the family that newborn jaundice may be exaggerated in its peak
bilirubin levels as well as in its duration
Neurology Answer 16
C. Reassure the family that this never results in any long-term complications of
the cranium or brain
The collection of fluid/blood in this image appears to be demarcated by the
suture lines and thus represents a cephalohematoma. In an otherwise well-
appearing infant, newborn care can continue in the Newborn Nursery. The
cephalohematoma can act as a reservoir of blood cells that during the resolution
phase can contribute to increased bilirubin levels and the potential need for
phototherapy. Resolution may take up to a month or more. And, although often
benign, there is a remote risk for infection and for skull deformation/brain
compression from significant calcification due to subpericranial osteogenesis.
(Please see Neurology Question 1 for additional details of skull findings related
to the birth process.)
Neurology Question 17
A Pediatrician examines a newborn and notices a deep dimple in the sacral
region with an adjacent large tuft of hair. The infant is otherwise well. The
Pediatrician orders a spinal ultrasound with one view shown below:

Of the following, the most appropriate NEXT STEP based on this ultrasound is:
A. An MRI of the infant’s entire spine and urological testing to assess
urologic function
B. Emergent surgery to avoid neurological injury
C. Repeat ultrasound when the child begins to walk to assess if the tethering
persists
D. Routine care as the spine ends in a normal location
Neurology Answer 17
A. An MRI of the infant’s entire spine and urological testing to assess urologic
function
The preliminary diagnosis of a tethered cord should be considered when the
end of the spine (i.e., conus medullaris) is located below the lumbar 1-2 disk
space. In the infant in this vignette the conus terminates at the midbody of
lumbar vertebrae 3, suggesting a tethered cord. The most appropriate next step
is to obtain a spinal MRI to confirm this diagnosis and assess for other spinal
abnormalities. Additional urological testing might identify urologic dysfunction
associated with a tethered cord. As a child grows, the tethering typically
becomes more severe because the spine becomes stretched. Surgery is typically
recommended if a child has clinical signs such as an abnormal gait, associated
pain/extreme irritability, loss of muscle function, and/or changes in bladder or
bowel function.
Reference: Khoury C. Closed spinal dysraphism: Clinical manifestations,
diagnosis and management. UpToDate. Accessed December 21, 2014.
Neurology Question 18
A pregnant woman meets with a pediatric neurologist to discuss the finding of
a fetal brain abnormality. The infant’s postnatal imaging is consistent with the
prenatal finding. One of the infant’s coronal ultrasonographic images is shown
below:

Of the following, the most likely diagnosis in this infant is:


A. Lissencephaly
B. Lobar holoprosencephaly
C. Polymicrogyria
D. Severe hydrocephalus
Neurology Answer 18
B. Lobar holoprosencephaly
This infant’s brain imaging shows a single fused monoventricle, which is
consistent with lobar holoprosencephaly. In this malformation, infants have an
incomplete separation of the 2 hemispheres of the brain. Radiographic findings
include:
Fusion of the frontal horns of the lateral ventricles
Wide communication with the 3rd ventricle
An absence septum pelluidum
Agenesis or hypoplasia of the corpus callosum
Presence of falx, the interhemispheric fissure and normal unfused thalamus
Lissencephaly is an abnormality of the number of gyral folds leading to a
“smooth” appearance of the brain. In contrast, polymicrogyria is caused by an
excessive number of gyri. An image of severe hydrocephalus, even if it is
obstructive, should still demonstrate a separation between the left and right side
of the brain structures.
Reference: Luijkx T, Gaillard F, et al. Lobar holoprosencephaly. Available
at: http://radiopaedia.org/articles/lobar-holoprosencephaly Accessed on
December 21, 2014
Neurology Question 19
A full-term infant is admitted to the NICU at 24 hours of age with respiratory
distress. The infant’s chest radiograph is shown below.

A postnatal echocardiogram reveals a structurally normal heart, dilation of the


left ventricle, right atrium, right ventricle, and superior vena cava with mildly
decreased ventricular function. In addition, the echocardiogram reveals nearly
holodiastolic flow reversal in the thoracic descending aorta and transverse arch.
A head US with and without Doppler is obtained and a coronal view is shown
below:

Of the following, the most appropriate consultant needed for this infant is a
pediatric:
A. Cardiothoracic surgeon
B. Hematologist
C. Interventional radiologist
D. Neurologist
Neurology Answer 19
C. Interventional radiologist
The infant in this vignette most likely has congestive heart failure as a result
of an arterio-venous malformation. This infant’s head ultrasound show a well-
circumscribed anechoic focus in the midline of the posterior brain that has fast
flow evident by color Doppler imaging. These findings most likely are a result
of a Vein of Galen aneurysmal malformation. Because this infant has signs of
cardiac overload, medical treatment of the heart failure is necessary. If this is
unsuccessful, embolization is needed. Thus, the most appropriate consultations
for the infant in this vignette are a pediatric cardiologist, neurosurgeon, and an
interventional radiologist.
Reference: Toulgoad F, Lasjaunias P. Vascular malformations of the brain.
Handb Clin Neurol. 112:1043-1051
V. AMPLITUDE-INTEGRATED EEGs
Amplitude-Integrated EEGs Question 1
A female infant is born by stat Cesarean section at 40 weeks’ gestation
because of a concern for placental abruption. The infant emerges apneic and
requires bag-mask ventilation and intubation. Her heart rate initially is 40 beats
per minute prompting initiation of chest compressions and a placement of an
umbilical venous catheter for intravenous epinephrine. Her Apgar score is 1 at 1
minute of age, 3 at 5 minutes of age, and 4 at 10 minutes of age. She is
transported to the NICU in an off isolette. Her examination is concerning for
encephalopathy. An amplitude-integrated EEG (aEEG) is obtained with the
following results:

Of the following, the most likely interpretation of this infant’s aEEG is:
A. Normal tracing during sleep phase
B. Moderately abnormal tracing with discontinuous pattern
C. Severely abnormal tracing with burst-suppression pattern
D. Signs of seizure activity
Amplitude-Integrated EEGs Answer 1
B. Moderately abnormal tracing with discontinuous pattern
An aEEG shows a basic trend of the peak-to-peak amplitude that is obtained
from a single EEG channel. The aEEG tracing of a healthy term newborn has
the following elements:
Evidence of sleep/wake cycling
Upper margin > 10 microvolts (microV)
Lower margin > 5 microV
Limited variability (narrow band); this is measured by the difference
between the upper and lower margins
The aEEG can assess background activity, electrophysiologic maturity and
behavioral states, and seizure activity. The aEEG of the infant in this vignette is
consistent with a moderately abnormal tracing with a discontinuous pattern and
shows the following abnormalities:

There is limited sleep/wake cycling evident on this infant’s tracing. There is


no evidence of a burst-suppression pattern, which is often described as having a
toothcomb appearance. Cumulatively, the findings on this infant’s aEEG can be
observed in an infant with moderate or severe encephalopathy or after an infant
has received anticonvulsant, sedative, or neuromuscular blocking medications.
This tracing is not impacted by artifact as there is minimal impedance (bottom
panel in question image).
References: Cherian PJ, Swarte RM, Visser GH. Technical standards for
recording and interpretation of neonatal electroencephalogram in clinical
practice. Ann Indian Acad Neurol. 2009;12:58-70
Amplitude-Integrated EEGs Question 2
A medical student is rotating in the NICU for an afternoon. She observes the
NICU team resuscitate a term infant. The infant requires intubation, chest
compressions, and intravenous epinephrine. The neonatology fellow performs a
complete neurological examination with the student; the exam is concerning for
encephalopathy. The fellow explains the need to obtain an aEEG recording on
this infant and reviews the procedure with the medical student.
Based on the electrode placement for a conventional EEG (shown in the
image below), which electrodes are used in an aEEG recording?

A. Bifrontal electrodes (7,9)


B. Bioccipital electrodes (12, 13)
C. Biparietal electrodes (4, 6)
D. Bitemporal electrodes (18, 19)
Amplitude-Integrated EEGs Answer 2
C. Biparietal electrodes (4, 6)
A full montage conventional EEG can use at least 21 electrodes (19 shown on
the diagram in the vignette and an additional 5 for the nose, ears, and back of the
head) and display 16 channels. This can be modified in neonates by using 9
electrodes as shown in the next diagram (electrode locations are displayed with
black circles and designated as 1,2,3,10,11,12,13,16,17).


A neonatal aEEG uses a single EEG channel, typically the left and right
parietal electrodes (shown below as circles 4 and 6).

This pair was selected to avoid artifact from facial muscles and movement
and to minimize interference with routine care. In addition, this location is ideal
for detecting abnormalities resulting from diffuse systemic hypoperfusion
because the electrodes are located over the apex of vascular watershed regions.
Recently, the addition of central electrodes (circles 2 and 3 in the diagram)
has been suggested as it may increase sensitivity and detect asymmetry (by
comparing recording of 2 and 4 with recording of 3 and 6). Use of bifrontal
electrodes (electrodes at 10 and 11 in the diagram) has sometimes been used to
avoid scalp hair but this practice is not recommended in neonates as neonatal
seizures are not typically located in this region and the frontal regions of a
neonate’s brain are not well developed.
Reference: El-Dib M, Chang T, Tsuchida TN, Clancy RR. Amplitude-
integrated electroencephalography in neonates. Pediatr Neurol. 2009;41:315-
326
Amplitude-Integrated EEGs Question 3
A neonatology fellow is reviewing the results of an aEEG of a term infant
with the pediatric residents (shown below). The fellow explains the difference
between the area designated as A and the area designated as B.

Of the following, the most likely interpretation of Section B is that it represents:


A. Active sleep
B. Movement or artifact
C. Quiet sleep
D. Seizure activity
Amplitude-Integrated EEGs Answer 3
Answer: C. Quiet sleep
Healthy term newborns have sleep/wake cycling that is characterized on an
aEEG by variations in bandwidths (peak-to-peak amplitudes). The area
designated as Section B in the aEEG of the infant in this vignette reveals a
slightly broader bandwidth at a lower amplitude, which is observed in quiet
sleep. The area designated as Section A has a slightly narrower bandwidth with
a higher amplitude, which is found when an infant is in active sleep or awake.
The presence of sleep/wake cycling is a normal finding in term infants. The
absence of sleep/wake cycling on a term infant’s EEG is abnormal; in contrast,
healthy preterm infants less than 30 weeks’ gestation may not have evidence of
sleep/wake cycling.
Reference: El-Dib M, Chang T, Tsuchida TN, Clancy RR. Amplitude-
integrated electroencephalography in neonates. Pediatr Neurol. 2009;41:315-
326
Amplitude-Integrated EEGs Question 4
4a. A pregnant woman at 39 ½ weeks’ gestation is admitted for evaluation
after she reports decreased fetal movements in the past 24 hours. A biophysical
profile is 2 out of 8 (2 points given for amniotic fluid volume) and a nonreactive
non-stress test. Because of lack of variability on the fetal heart rate monitor, the
obstetrical team delivers the infant emergently by Cesarean section.
The infant emerges apneic and is hypotonic with a good heart rate. After
several minutes of bag-mask ventilation, the infant has an improved respiratory
effort but remains hypotonic. Due to intermittent apnea, the NICU team
intubates the infant at 5 minutes of age and then transports the infant to the
NICU in an off warmer. Upon arrival to the NICU, the team obtains an aEEG,
which is shown below:

Of the following, the most likely interpretation of this infant’s aEEG is:
A. Normal tracing during sleep phase
B. Moderately abnormal tracing with discontinuous pattern
C. Severely abnormal tracing with suppression pattern
D. Signs of seizure activity
4b. The aEEG of this infant continues for 45 additional minutes (see
following aEEG). The pediatric resident providing night coverage reviews the
aEEG and contacts the neonatology fellow because of the dramatic change in the
infant’s tracing (shown in the red oval).

Of the following, the most likely explanation for this aEEG finding is:
A. Active sleep
B. Movement or artifact
C. Quiet sleep
D. Seizure activity
Amplitude-Integrated EEGs Answer 4
4a. C. Severely abnormal tracing with suppression pattern
The infant in this vignette has a severely abnormal tracing because of the
following:
No evidence of sleep/wake cycling
Upper margin less than 10 microV
Greatly reduced variability (narrow band, i.e., small difference between the
maximum and minimum peak-to-peak amplitudes)
The finding of amplitude suppression on an aEEG could result from hypoxic-
ischemic encephalopathy (HIE). However, artificially reduced amplitude can be
caused by scalp edema or a cephalohematoma because these findings increase
the space between the recording electrodes and the brain. An artificially narrow
bandwidth can be found in patients with HIE but may also be an artifact of a
high-frequency ventilator (high lower margin) or seizures (high lower and upper
margins). Thus, aEEG interpretation warrants careful consideration of other
factors.
Reference: El-Dib M, Chang T, Tsuchida TN, Clancy RR. Amplitude-
integrated electroencephalography in neonates. Pediatr Neurol. 2009;41:315-
326
4b. B. Movement or artifact
The aEEG recording of the infant in this vignette designated in the oval
region shows higher upper and lower margins compared with the rest of the
recording. As this might represent seizure activity, it would be helpful to assess
the EEG in real time. However, the real-time EEG shown in the middle panel is
recording the cerebral activity at 18:05 (dotted line) and thus, is not helpful in
interpreting the recording in the oval region. To help interpret the aEEG
findings in the oval region, the neonatology fellow can scroll to this precise time
point and assess the raw EEG reading.
Within the oval section, there is a dotted green line at 20:16:30 in the upper
panel (marked as “A”). The bedside clinician has recorded that an examination
is being performed at that exact time (lower left side with designation “20:16:30
Exam”). In addition, the impedance (lower panel) is increased during the time
of this exam. These data suggest artifact or that the infant is moving and thus,
the aEEG recording in the oval region is not reliable. The following aEEG
summarizes this interpretation.

Amplitude-Integrated EEGs Question 5
A 30-year old pregnant woman at 40 ½ weeks’ gestation contacts her
obstetrician because she is concerned about lack of fetal movements in the past
24 hours. Her obstetrician suggests that she come to the hospital immediately
for an evaluation. After a failed oxytocin contraction challenge, the obstetrician
delivers the infant by Cesarean section. There are no concerns for infection.
The baby boy emerges hypotonic, pale, and has a brief cry followed by
apnea. His heart rate is always above 100 beats per minute. The NICU team
provides positive pressure ventilation for 3 minutes and this is discontinued
when he starts having spontaneous respirations. His Apgar score is 4 at 1
minute, 6 at 5 minutes, and 7 at 10 minutes. He is admitted to the NICU for
further evaluation.
Shortly after arrival to the NICU, the baby has several episodes of
desaturations prompting intubation. His cord arterial blood gas is: pH=6.88,
pCO2=32 mm Hg, pO2=28 mm Hg with a base excess of -29 mEq/L. His
examination is significant for extreme pallor and symmetric hypotonia. His
hematocrit is 9%.
Amplitude-integrated EEG leads are placed on the infant and the recording is
shown below:
Of the following, the most likely interpretation of this infant’s aEEG is:
A. Normal tracing
B. Moderately abnormal tracing with discontinuous pattern
C. Severely abnormal tracing with suppression pattern
D. Signs of seizure activity
Amplitude-Integrated EEGs Answer 5
D. Signs of seizure activity
Electrographic seizures in a neonate are characterized by an abrupt, transient,
sharp increase in the lower margin of the aEEG, typically with a corresponding
smaller rise in the upper margin. The bandwidth during the period of the seizure
is narrower than baseline. While a conventional EEG can be useful at
identifying the precise location of the seizure, an aEEG recording cannot
determine the anatomical site of origin of the seizure because it uses a limited
number of electrodes. For this same reason, aEEGs are not as sensitive at
detecting seizures as conventional EEGs and usually underestimate the number
of seizures. Because patient manipulation can lead to artifact that can be
mistaken for seizures, any intervention should be noted by the bedside clinician
during aEEG recordings. If there is a concern for seizure activity based on an
infant’s aEEG recording, the corresponding raw EEG tracing needs to be
reviewed.
The aEEG of the infant in this vignette shows evidence of seizure activity
(highlighted in red ovals on the following aEEG) lasting between 2 and 15
minutes. The impedance is not significant during these episodes and thus,
artifact is unlikely. The black dotted line in the upper panel reflects the time of
22:13:45 when the infant’s raw EEG is recorded (shown in the middle panel).
This raw EEG shows repetitive waveforms that are consistent with seizures.
This infant requires a conventional EEG to identify the location of the seizure
activity.
Reference: El-Dib M, Chang T, Tsuchida TN, Clancy RR. Amplitude-
integrated electroencephalography in neonates. Pediatr Neurol. 2009;41:315-
326
The authors would like to thank Dr. Deirdre O’Reilly (neonatologist,
Children’s Hospital, Boston) for her thorough editing of this chapter.
VI. GASTROENTEROLOGY
Gastroenterology Question 1
A 3-week old female infant born at 28 weeks’ gestation has a bilious spit.
The infant’s abdominal radiograph is shown below.

Of the following, the most appropriate NEXT step in the management of this
infant is:
A. Obtain a contrast enema
B. Obtain an abdominal ultrasound
C. Placement of a peritoneal drain
D. Stop feeding and start antibiotics
Gastroenterology Answer 1
D. Stop feeding and start antibiotics
The abdominal radiograph of the infant in this vignette shows multiple loops
of intramural air, particularly apparent in the right lower quadrant (shown
below).

There is no evidence of intraperitoneal air and thus, surgery is not indicated.


Rather, the infant requires bowel rest and initiation of antibiotics.
Gastroenterology Question 2
A term infant has cyanosis in the delivery room and a large amount of oral
secretions. The infant’s chest and abdominal radiograph at 4 hours of age is
shown below.

Of the following, the most likely variant of esophageal atresia (EA) in this infant
is:
A. EA with fistula from distal esophagus to trachea
B. EA with fistula from proximal esophagus to trachea and fistula from distal
esophagus to trachea
C. EA with upper pouch fistula
D. H-type fistula
Gastroenterology Answer 2
C. EA with upper pouch fistula
The radiograph of the infant in this vignette shows that there is an absence of
air in the infant’s stomach and intestines. Typically gas is present in the small
intestines by approximately 3 hours of age. The coiling of the naso/orogastric
tube in the esophagus correlates with an esophageal obstruction. Infants with an
esophageal atresia with an upper pouch fistula or without any fistula will have an
absence of air entering the gastrointestinal tract. Affected infants will have a
scaphoid abdomen. The Table below compares the 5 different types of
tracheoesophageal abnormalities.

Reference: Printed with permission from: Brodsky D, Martin C. Neonatology


Review. 2nd edition. Lulu. 2010
Gastroenterology Question 3
An infant born at 27 weeks’ gestation has acute abdominal distention. The
infant’s abdominal radiograph is shown below:

Of the following, the most likely diagnosis in this infant is:


A. Duodenal atresia
B. Hirschsprung’s disease
C. Necrotizing enterocolitis
D. Septic ileus
Gastroenterology Answer 3
C. Necrotizing enterocolitis
This infant’s abdominal radiograph demonstrates a massive
pneumoperitoneum. The bowel gas pattern is non-obstructive with intraluminal
air through to the rectum. Although there is no convincing evidence for
pneumatosis intestinalis, necrotizing enterocolitis is the most likely cause of this
infant’s perforation of the choices given. Of note, this infant’s umbilical venous
catheter tip overlies the medial aspect of the right 11th rib and needs to be
removed.
Gastroenterology Question 4
A full-term infant is admitted to the NICU with a prenatal diagnosis of a
gastrointestinal anomaly. The infant’s abdominal film is shown below.


Of the following, the most likely cause of this infant’s anomaly is:
A. Cytomegalovirus infection during the first trimester
B. Failure of recanalization of intestinal portion
C. Intestinal torsion during the third trimester
D. Intrauterine vascular accident
Gastroenterology Answer 4
B. Failure of recanalization of intestinal portion
The abdominal radiograph of the infant in this vignette shows a collection of
air in the stomach and proximal intestine. This “double bubble” radiographic
finding is most consistent with duodenal atresia. During normal intestinal
development, the duodenum becomes occluded by proliferating epithelial cells
during the 5th and 6th weeks of gestation. Following, the hollow duodenum is
reestablished over the next several weeks as the duodenum becomes
recanalized. If this recanalization phase does not occur, duodenal stenosis or
atresia ensues. In contrast, jejunoileal atresia results from an intrauterine
ischemic injury to a region of a normally formed intestine.
Reference: Sadler TW, Langman J. Digestive system. Langman’s Medical
Embryology. 11th ed. Philadelphia, Pa: Lippincott Williams & Wilkins,
2006:203-229
Gastroenterology Question 5
A 25-week gestational age male infant has feeding intolerance and a slightly
distended abdomen. His abdominal radiograph is shown below.

All of the following findings are evident on this film EXCEPT for a(n):
A. Absence of distal air
B. Obstructive gas pattern
C. Pneumoperitoneum
D. Thickened bowel loop walls
Gastroenterology Answer 5
B. Obstructive gas pattern
The AP abdominal radiograph of this infant shows a lucency projecting over
the abdomen that is consistent with a large amount of intraperitoneal air. The
wall of the intestinal loops appears thickened.

Although there is an absence of distal air, the visualized intestinal loops are not
dilated and thus, an obstructive pattern is not apparent.
Gastroenterology Question 6
A term infant in the delivery room has worsening cyanosis despite positive
pressure ventilation.

The following clinical findings are supportive of this infant’s diagnosis


EXCEPT:
A. Absent bowel sounds
B. Heart tones shifted to the right
C. Polyhydramnios
D. Scaphoid abdomen
Gastroenterology Answer 6
A. Absent bowel sounds
This infant’s radiograph shows a left-sided congenital diaphragmatic hernia.
Although bowel sounds may not be in the typical location, they should still be
heard in an infant with a diaphragmatic hernia. Due to the bowel contents in the
thoracic cavity, heart tones will be shifted to the right in infants with a left-sided
hernia. Affected infants typically have a scaphoid abdomen and polyhydramnios
is commonly present prenatally.
In another example radiograph, the course of the NG tube illustrates the
degree of gastrointestinal displacement that can occur in infants with a
congenital diaphragmatic hernia.
Gastroenterology Question 7
A Pediatric Resident is called to assess a 12-hour old full-term newborn with
persistent, non-bilious emesis. An abdominal radiograph is obtained:

The best explanation for this radiographic finding is a(n):


A. Absence of distal air may be normal at this early postnatal age
B. Anatomical abnormality is causing complete gastric outlet obstruction
C. Early presentation of severe pyloric stenosis
D. Gastric mass or cyst is preventing normal gastric flow
Gastroenterology Answer 7
B. Anatomical abnormality is causing complete gastric outlet obstruction
The only pocket of abdominal air in this radiograph represents the gastrum.
The absence of any distal air suggests a complete obstruction (versus
incomplete). Complete obstruction can be caused by a gastric atresia or by a
pyloric web. Pyloric stenosis, a gastric mass, or a gastric cyst tends to cause an
incomplete obstruction. By 12 hours of age, in a normal infant, bowel gas should
be present throughout the small bowel.
Gastroenterology Question 8
A late preterm male infant is born to an 18-year old G1P0 woman. Prenatal
serologies were unremarkable. Except for a cold in the 8th week of pregnancy for
which the mother took oral pseudoephedrine and nasal steroids, the pregnancy
was unremarkable. No other fetal anomalies were found on full fetal assessment.
The infant’s radiograph is shown below.

The infant’s parents are asking about recurrent risk with future pregnancies and
the preferred response is:
A. As high as 10%
B. Less than 0.05%
C. Risk is sex-specific
D. Undetermined and would require full genetic screening of both parents
Gastroenterology Answer 8
D. Undetermined and would require full genetic screening of both parents
The abdominal radiograph shows the shadow of a silo that is used for staged
repairs of abdominal wall abnormalities, such as an omphalocele and a
gastroschisis. Several variables present in this vignette suggest that the defect
depicted in this radiograph is a gastroschisis: young maternal age, use of
pseudoephedrine, presence during the first trimester, and lack of other fetal
anomalies.
Overall, the recurrent risk of gastroschisis is thought to be around 2% to 3%.
However, this may be underestimated given reports of familial occurrences
including an autosomal dominant variant. Thus a complete genetic consultation
should be provided before estimates of recurrences are provided to families.
Gastroenterology Question 9
A liver ultrasound is performed in a 2-day old full-term infant with
cholestasis. One ultrasonographic image is shown below:

All of the statements below are consistent with this finding, except:
A. A calcified object is present within a dilated bile duct.
B. The infant is not at an increased risk of having cystic fibrosis.
C. This is a common prenatal ultrasonographic finding with an estimated
incidence of 1:1,000 to 1:1,750.
D. This is an incidental finding and can be a result of a fetal viral infection.
Gastroenterology Answer 9
A. A calcified object is present within a dilated bile duct.
Intrahepatic calcifications are a common prenatal radiographic finding. They
may represent abnormalities of the fetal liver including calcified thrombi and
parenchymal injury from viral infections. The location of the calcification cannot
be accurately determined by ultrasound (parenchyma versus bile duct versus
portal vein). There are no reports associating this finding with cystic fibrosis.
Gastroenterology Question 10
A mildly dysmorphic, 3-day old full-term infant fails to pass meconium. An
initial evaluation reveals the following abdominal radiograph:

Reasonable interpretations and next steps include all of the following, except:
A. A barium enema should be arranged.
B. An evaluation of associated physical findings may aid in narrowing the
diagnostic possibilities.
C. Given the lack of air in the rectal area, a digital exam should be performed
to check for the presence of stool in the rectal vault.
D. The abdominal radiograph suggests a distal obstructive pattern.
Gastroenterology Answer 10
C. Given the lack of air in the rectal area, a digital exam should be performed to
check for the presence of stool in the rectal vault.
The abdominal radiograph of this infant suggests an obstructive pattern; a
distal obstruction is more likely than a proximal obstruction given the high
number of visualized loops of bowel. A barium enema is a reasonable next step
to determine the diagnosis. The hint of dysmorphic features in the clinical
vignette points to Hirschsprung’s disease, where 20% of cases are in conjunction
with other congenital defects.
A rectal digital exam should be avoided in an infant with suspected
Hirschsprung’s disease. This procedure may diminish the caliber change
between the normal and abnormal innervated areas of the colon, eliminating the
transition zone seen with a barium enema that can be a hallmark feature of this
disease.
Gastroenterology Question 11
This film is very reminiscent of Case#10. Besides Hirschsprung’s disease, list
5 additional diagnoses that may present with a distal intestinal obstructive
pattern.

(1) __________________________________
(2) __________________________________
(3) __________________________________
(4) __________________________________
(5) __________________________________
Gastroenterology Answer 11
Colonic atresia
Ileal atresia
Jejunal atresia
Malrotation/volvulus
Meconium ileus
Meconium plug
Small left colon syndrome
Gastroenterology Question 12
A full-term infant is admitted to the intensive care unit because of a physical
examination finding of a tense, distended abdomen. The infant is otherwise
stable without the need for respiratory or cardiovascular support. A nasogastric
tube is placed to decrease the infant’s gastrointestinal distention and minimal
clear fluid is returned. The infant has passed meconium.

Of the following, the most accurate statement is:


A. The clinical presentation and abdominal radiograph do not suggest an
obstructive pattern of the gastrointestinal tract.
B. The infant has a greater than 50% chance of having cystic fibrosis.
C. The organ system that is the most common cause for this presentation is
renal.
D. Surgical intervention is likely.
Gastroenterology Answer 12
D. Surgical intervention is likely.
The abdominal radiograph of this infant is consistent with neonatal ascites –
the bulging flanks suggest distention and the centralized loops of bowel suggest
peritoneal fluid.
Intestinal obstruction should always be considered with an abnormal
distribution of bowel gas. In neonatal ascites as a general class, renal
abnormalities are the most likely cause. However, if one looks closely,
calcifications can be seen on this radiograph (shown by the blue arrows in the
image below), shifting the differential diagnoses to the area of meconium
peritonitis.

For meconium peritonitis, bowel etiologies are most likely. And, although
meconium ileus is in this differential, cystic fibrosis accounts for less than 50%
of all meconium peritonitis cases. It is likely that surgical intervention will be
needed for the underlying bowel abnormality.
Gastroenterology Question 13
An abdominal radiograph is obtained in a full-term infant with feeding
intolerance.

Diagnoses that are consistent with this radiograph include all of the below,
except:
A. Diaphragmatic hernia
B. Esophageal atresia with an upper pouch fistula
C. Gastric atresia
D. Isolated esophageal atresia
Gastroenterology Answer 13
C. Gastric atresia
A gasless abdomen is typically an ominous finding. Gastric atresia is
obstruction at the outlet, not inlet and thus, infants with gastric atresia typically
have a radiographic finding of a large gastric bubble. Esophageal atresia with an
upper pouch proximal fistula to the trachea can lead to this radiographic
appearance. A diaphragmatic hernia can also result in an apparent gasless
abdomen due to the shifting of abdominal contents into the thoracic cavity.
Gastroenterology Question 14
The Fellow is called to the bedside of a preterm infant with abdominal
distention and inconsolable crying. On exam, the abdomen is distended and
slightly firm. Active bowel sounds are present. An abdominal radiograph is
obtained:

Of the following, the preferred NEXT step in the management of this infant is:
A. Call Pediatric Surgery to evaluate the need for surgical intervention
B. Obtain a cross-table or decubitus abdominal radiograph to rule out a
pneumoperitoneum
C. Place a replogle, make NPO, obtain a CBC and blood culture, and begin
broad spectrum antibiotics
D. Reassure the nurse that the findings are normal
Gastroenterology Answer 14
A. Call Pediatric Surgery to evaluate the need for surgical intervention
This clinical vignette depicts an infant with an incarcerated hernia. The
portable AP abdomen shows moderate nonspecific distention of multiple loops
of bowel with gas. A loop of bowel is seen in the left hemiscrotum. Lung bases
show moderate interstitial lung disease. If the inguinal hernia cannot be reduced
at the bedside, the surgical service needs to be called for possible surgical
intervention.
Gastroenterology Question 15
A 2-month old former 29 week gestational age male infant presents with
persistent, projectile vomiting. A 1-month of age he had a course of oral
erythromycin to improve feeding tolerance with no clear benefit. On further
family history, the mother reports that she had pyloric stenosis as an infant. An
abdominal ultrasound is obtained:

All of the statements below are false regarding this clinical vignette, except:
A. An upper GI barium study would reveal the classic “apple core” image.
B. Hyperchloremic, metabolic alkalosis is the typical metabolic aberration.
C. The risk in offspring is greatest if the father had this same diagnosis.
D. This condition is five times more likely in males than females.
Gastroenterology Answer 15
D. This condition is five times more likely in males than females.
The infant in this vignette has pyloric stenosis (PS). The incidence of PS is
approximately 3 in 1,000 births and males are 5x more likely than females to
have this condition. The risk is also greatest in the first-born male. The
recurrence risk varies based on the familial pattern:
If mother with pyloric stenosis =19% risk of affected son, 7% risk of
affected daughter
If father with pyloric stenosis = 5.5% risk of affected son, 2.4% risk of
affected daughter
If one child with pyloric stenosis = 3% risk to next child (4% if male, 2.4%
if female)
The classic metabolic pattern in an infant with PS is a hypochloremia, metabolic
alkalosis; and the classic barium enema sign is the “string sign”. The ultrasound
image in this case depicts two important features: (1) an elongated pylorus; and
(2) and thickened pylorus.

Reference: Brodsky D, Martin C. Neonatology Review. 2nd edition. Lulu.


2010
Gastroenterology Question 16
A 2-month old, former 26 week gestational age infant, has repeated bouts of
abdominal distention and non-specific abdominal radiographic findings. A
barium study is conducted.

All of the following clinical scenarios are possible for this infant EXCEPT for:
A. A diagnosis of Hirschsprung’s disease
B. At 4 weeks of age the infant had NEC that required surgical intervention
C. At 4 weeks of age the infant had NEC that was medically managed
D. Previous repair of jejunal atresia
Gastroenterology Answer 16
A. A diagnosis of Hirschsprung’s disease
There is an approximate 50% risk of a post-necrotizing enterocolitis stricture,
with higher rates in surgically treated infants. The most common location for a
stricture is the distal ileum in infants with surgically treated NEC versus a
colonic location in infants with medically managed NEC.
Previous abdominal surgery is a risk factor for strictures.
Although it would be unusual to present with Hirschsprung’s at two months
of age, a late diagnosis is possible. However, infants with this disease will have
barium enema studies that reveal a smaller caliber rectum compared to the
proximal colon.
Gastroenterology Question 17
The neonatal team is carefully monitoring a 26 week gestational age infant
with abdominal distention. Vital signs are stable and except for diffusely dilated
loops of bowel, there is no evidence for necrotizing enterocolitis. The call team
decides to take an overnight abdominal radiograph but otherwise continue
present management. The next morning the Attending asks the Fellow how
things went during the shift. The Fellow informs the Attending that the infant is
much improved with reduced distention and bowel loops normalizing in caliber.
The Attending reviews the film with the Fellow:

Of the following, the preferred NEXT step in the management of this infant is to:
A. Obtain an abdominal ultrasound to rule out NEC
B. Order another radiograph before morning rounds to aid in the decision to
start trophic feedings
C. Repeat the abdominal radiograph with 2-views
D. Share in relief with the Fellow that the infant’s clinical status has improved
Gastroenterology Answer 17
C. Repeat the abdominal radiograph with 2-views
Looking closely at this infant’s radiograph, there are subtle clues to a
diagnosis of a large pneumoperitoneum, which deceivingly can initially present
with reduced abdominal distention and smaller caliber of bowel loops after
release of the air pressure. Findings include: (1) Clear outline of bowels loops
with air on both sides (not shown on this infant’s radiograph); (2) outline of liver
falciform ligament (not shown on this infant’s radiograph); (3) air outlining flank
edge (blue arrows on this infant’s radiograph below); and (4) presence of air not
within the gastrum or bowel loops (red arrows on this infant’s radiograph
below).

A second view, such as a right-side up lateral decubitus film as shown below,


can help clarify the presence of air. In this case the liver is used to provide a nice
contrast and edge between bowel contents and air.
VII. RENAL AND GENITOUINARY
Renal and Genitourinary Question 1
A full-term, large for gestational age male is born by emergent Cesarean
section for prolonged fetal bradycardia. He requires positive pressure ventilation
in the delivery room with subsequent intubation and 0.50 inspired fraction of
oxygen. In the intensive care unit, he is transferred to the ventilator and
umbilical lines are placed. Mean blood pressures and urine output remain low
for age despite several normal saline boluses and initiation of vasopressors. On
exam, a palpable mass is noted in the left abdominal flank. A renal ultrasound is
obtained and one view is shown below:

Management of this infant may include all of the following except:


A. Check hematocrit level anticipating the potential need for a packed red
blood cell transfusion
B. Measure cortisol levels
C. Order for urine catecholamine levels
D. Provide parenteral nutrition with standard amounts of potassium and above
standard amounts of sodium and glucose
Renal and Genitourinary Answer 1
D. Provide parenteral nutrition with standard amounts of potassium and above
standard amounts of sodium and glucose
A large, thin-walled cyst is identified adjacent to and inferiorly compressing
the upper pole of the right kidney. The cyst appears to be located within the right
adrenal gland and contains multiple internal septations. Examination with color
Doppler reveals no internal vascularity. This cyst may represent a resolving
adrenal hemorrhage or a cystic neuroblastoma.

With adrenal hemorrhage, a significant amount of total blood loss may


require a transfusion. Additionally, impaired cortisol responses may be a
consequence of adrenal injury. Urine catecholamine levels will assist in
differentiating a neuroblastoma (which will be positive) versus adrenal
hemorrhage.
With adrenal impairment, electrolyte disturbances may result including
hypoglycemia, hyponatremia, and hyperkalemia. Provision of exogenous
potassium should be done judiciously.
Renal and Genitourinary Question 2
The Attending Neonatologist is asked to meet prospective parents with an
abnormal genitourinary finding. An amniocentesis is consistent with a female
fetus. The following fetal image is obtained:

Appropriate discussion points include all of the following except:


A. If isolated, good clinical and developmental outcomes are expected.
B. Large lesions can lead to pulmonary hypoplasia.
C. The infant should be examined for commonly associated findings in other
organ systems including renal, cardiac, and cerebral.
D. This finding represents a fluid-filled vaginal and uterine cavity secondary
to an obstruction process.
Renal and Genitourinary Answer 2
C. The infant should be examined for commonly associated findings in other
organ systems including renal, cardiac, and cerebral.
The fetal image demonstrates a hydrometrocolpos – a fluid-filled enlargement
of both the vaginal cavity and uterus secondary to obstruction. If just the vaginal
cavity were enlarged it would be termed a hydrocolpos. Obstructive causes
include vaginal web, vaginal atresia, cloacal abnormalities, or imperforate
hymen. If isolated, the outcome is generally good. Although associated findings
can be found in the urogenital, gastrointestinal and cardiac systems, associated
cerebral findings are rare.
Renal and Genitourinary Question 3
An infant has an abdominal ultrasound with one image shown below:

The clinical scenario that best fits this image is a:


A. Preterm infant on Lasix for severe bronchopulmonary dysplasia
B. Preterm infant with a history of central umbilical lines and now with a
renal vein thrombosis
C. Term infant with inherited cystic renal disease and high likelihood for
early dialysis
D. Term infant with moderate hydronephrosis on antibiotic prophylaxis
Renal and Genitourinary Answer 3
A. Preterm infant on Lasix for severe bronchopulmonary dysplasia
This image depicts nephrocalcinosis with two major areas of multiple bright,
echogenic spots. This is a common sequelae of prolonged diuretic use. There is
no evidence of thrombosis. The kidney appears otherwise normal with normal
calices, thus there is no evidence for abnormal cysts or hydronephrosis.
Renal and Genitourinary Question 4
A former 25 week gestational age preterm infant, now 5 days old, presents
with acidosis and oliguria. A Doppler renal ultrasound reveals the following
image:

Of the following, the preferred NEXT step in the management of this infant is:
A. Begin heparin
B. Do nothing, the study is normal
C. Obtain a repeat renal ultrasound in two weeks
D. Perform an echocardiogram
Renal and Genitourinary Answer 4
D. Perform an echocardiogram
A schematic of normal arterial blood flow velocity as determined by Doppler
is shown below. Both systolic and diastolic phases should be above zero
indicating normal forward velocity.


This Doppler study of the kidney of the infant in this vignette demonstrates
reversed diastolic blood flow of the renal artery. This is due to reduced blood
flow, which can result from a hemodynamically significant patent ductus
arteriosus (PDA) shunting blood away from the renal arteries or a renal artery
thrombosis. Even if the cause of this finding is due to a thrombosis, heparin
therapy is often not required. This is an abnormal study so it is necessary to
repeat the image in a shorter period of time than 2 weeks. Given the potential
contribution of the blood flow pattern in symptomatic PDAs, an echocardiogram
would be the preferred next step of the options presented.
Renal and Genitourinary Question 5
A prenatal ultrasound in a female fetus reveals the following image of a
kidney.

Appropriate management of this infant includes all of the following, except:


A. Antibiotic prophylaxis
B. A voiding cystourethrogram (VCUG) in 1 month
C. Postnatal renal ultrasound within 24 to 48 hours
D. Routine care and monitoring in the Newborn Nursery
Renal and Genitourinary Answer 5
C. Postnatal renal ultrasound within 24 to 48 hours
This image reflects caliectasis or hydronephrosis. Antibiotic prophylaxis is
typically given to infants with moderate to severe hydrocephalus. An early
follow-up renal ultrasound after birth can be falsely negative given the diuretic
phase (with weight loss) that occurs after delivery. Thus, the best time for a
follow-up study is at approximately 1 month. Typically a VCUG is also
performed at one month. Finally, it is appropriate for the infant to receive
routine care in the Newborn Nursery.
Renal and Genitourinary Question 6
The covering night-time Pediatric Resident is called to the Nursery to assess the
genitourinary exam in a full-term male. On exam, the infant’s left testicle is
barely palpable within a tense, firm scrotal sac. The right testicle feels, in
comparison, larger and the scrotal sac has a bluish discoloration.

Of the following, the preferred NEXT step in the management of this infant is:
A. Arrange for an Urology consultation first thing in the morning
B. Call Urology for immediate exploration of the left testicle
C. Call Urology for immediate exploration of the right testicle
D. Reassure the family that the physical findings are within a normal range
Renal and Genitourinary Answer 6
C. Call Urology for immediate exploration of the left testicle
Evaluation and management for a possible testicular torsion is an emergency
(should not be delayed till the morning) and this entails coordination of care with
Urology with possible surgical exploration. The testicle on the left shows an
absence of blood flow and is enlarged. This is not a normal presentation. In
contrast, the testicle on the right demonstrates blood flow and is accompanied by
a large hydrocele. This can be a normal newborn finding.
Renal and Genitourinary Question 7
A term infant has a renal ultrasound with one view shown below:

Correct statements about the diagnosis and outcome of this infant includes all of
the following except:
A. Cysts are small and scattered throughout the renal parenchyma, and are
likely to include both kidneys.
B. It is more common to have unilateral involvement.
C. Renal function and blood pressures need to be monitored closely.
D. The risk for recurrence in a subsequent child is low.
Renal and Genitourinary Answer 7
A. Cysts are small and scattered throughout the renal parenchyma, and are likely
to include both kidneys.
The hyperechoic appearance to the kidney is indicative of underlying disease.
This, in conjunction with the presence of peripheral cysts, suggests multicystic
dysplasia. For this diagnosis, statements B, C, and D are correct. The other major
diagnosis that has renal cysts is polycystic kidney disease (PCKD), which can be
autosomal dominant or recessive. Statement A best reflects PCKD. This renal
disease is typically bilateral and the risk of recurrence is higher than that of
multicystic disease. Close monitoring of renal function and blood pressures are
true of both classes of cystic renal disease.
Renal and Genitourinary Question 8
Prenatal imaging of a male fetus in the 21st week of gestation demonstrates
the two images below (left bladder; right kidney). In addition, imaging of the
amniotic fluid pockets is consistent with oligohydramnios.

Prenatal consultation with the family should included the following discussion
points EXCEPT:
A. Oligohydramnios present in the second trimester has an increased perinatal
mortality, as high as 90 to 95 percent.
B. Success rates after fetal surgery are excellent and it is recommended that
the family pursues this option.
C. The ultrasound findings that support this diagnosis are bilateral
hydronephrosis, dilated bladder, and a dilated posterior urethra – the latter
together giving a “keyhole” appearance.
D. This is caused by the persistence of the urogenital membrane.
Renal and Genitourinary Answer 8
B. Success rates after fetal surgery are excellent and it is recommended that the
family pursue this option
This is a case of posterior urethral valves, the most common cause of urinary
obstruction in males. Statements A, C and D accurately describe this condition.
On the left is an image of a dilated bladder with a thickened wall (blue line); on
the right, severe hydronephrosis is seen. Since the obstruction is distal to the
kidneys and bladder, both kidneys are affected. The earlier in gestation that the
obstruction is present, the more severe the morbidity with high rates of mortality.

This morbidity and high mortality risk is largely driven by reduced amniotic
volume and impaired pulmonary development (pulmonary hypoplasia). This can
also be due to pulmonary compression from a severely enlarged bladder. Fetal
surgery began as early as 1981, however, the outcomes are very poor with high
mortality due to premature birth and respiratory failure. This option is now
typically reserved for the most severe cases of fetal obstruction and compromise
(absent to minimal amniotic fluid with likely secondary compromise to
pulmonary development).
Reference: Holmes N. Clinical presentation and diagnosis of posterior
urethral valves and Management of posterior urethral valves. In: UpToDate,
Baskin LS (Ed), UpToDate, Waltham, MA. (Accessed on December 21, 2014.)
Renal and Genitourinary Question 9
A 2nd trimester antenatal abdominal ultrasound is routinely obtained and one
of the renal images is shown below:

Of the following, the most correct statement for this image is:
A. No postnatal imaging is necessary.
B. This happens because of abnormal migration of one or both kidneys.
C. This is a rare condition averaging 1 per 10,000 births.
D. This is an incidental finding and, if isolated, no health consequences are
expected.
Renal and Genitourinary Answer 9
C. This is a rare condition averaging 1 per 10,000 births.
The image in this vignette illustrates a rare anomaly called a horseshoe
kidney. This occurs on average 1 per 10,000 births as a result of abnormal
migration of BOTH kidneys and resultant fusion at the lower poles (versus upper
90% of the time). When fusion occurs as a result of abnormal migration of one
kidney, this is called crossed fused renal ectopy.

Although an affected patient with a horseshoe kidney is generally asymptomatic,


complications can occur such as infection, renal calculi, and urinary obstruction.
After birth, a renal ultrasound is recommended to confirm anatomy, and if at any
point a urinary tract infection occurs, a VCUG should be performed.
Reference: Rosenblum ND. Renal ectopic and fusion anomalies. In:
UpToDate, Matoo TK and Baskin LS (Eds), UpToDate, Waltham, MA.
(Accessed on December 21, 2014.)
Renal and Genitourinary Question 10
A follow-up abdominal US for a female infant was performed to re-evaluate a
cystic mass noted on prenatal ultrasound. The left image below shows an ovarian
cyst compared to the right image, which outlines a normal ovary.

Of the following statements, the most INCORRECT response about the ovarian
cyst is:
A. By 6 months of age, a majority spontaneously resolved. Thus expectant management is
recommended with abdominal ultrasounds every few months.
B. Differential diagnosis broadly includes genitourinary tract anomalies and gastrointestinal tract
disorders.
C. Ovarian cysts are common, occurring in approximately 1 in 2,500 female births.
D. There are no known familial or maternal risk factors.
Renal and Genitourinary Answer 10
D. There are no known familial or maternal risk factors.
Statements A, B, and C are correct. Maternal risk factors have been
identified and include diabetes mellitus and preeclampsia.
VIII. GENETICS & SKELETAL SYSTEM
Genetics & Skeletal System Question 1
A term infant is born with short limbs. The infant’s extremity films are
shown below:

The measurements by the neonatologist, geneticist, and radiologist reveal that


both the proximal and distal bones of the arms and legs of the infant are short.
Of note, the bones of the infant’s feet and hands are not visualized on these films
but other radiographic images reveal that they are also small.
Of the following, the description that best describes this infant’s skeletal
abnormality is:
A. Acromelia only
B. Mesomelia without rhizomelia
C. Micromelia
D. Rhizomelia without acromelia
Genetics & Skeletal System Answer 1
C. Micromelia
This infant has micromelia (entire limb is small), rhizomelia (shortening of
proximal long bones), mesomelia (shortening of distal long bones), and
acromelia (short bones of hands and feet). This is summarized below:
Genetics & Skeletal System Question 2
A term infant with a prenatal diagnosis of trisomy 18 syndrome is admitted to
the NICU with respiratory distress. The infant’s chest radiograph is shown
below:

Of the following, the LEAST likely bone abnormality in infants with trisomy 18
is:
A. Eleven rib pairs
B. Hypoplastic ribs
C. Osteopenic long bones
D. Vertebral anomalies
Genetics & Skeletal System Answer 2
C. Osteopenic long bones
The chest radiograph of this infant reveals some common bone findings in an
infant with trisomy 18 syndrome. These are: eleven rib pairs, hypoplastic ribs,
and vertebral anomalies. Osteopenic long bones are not evident on this infant’s
film and are not associated with trisomy 18. Additional features of infants with
trisomy 18 includes:
Prominent occiput
Low-set malformed ears
Micrognathia
Clenched hand with tendency of overlapping of index finger over third
finger and fifth finger over fourth finger
Low arch dermal ridge patterning on fingertips
Short sternum
Small pelvis
Structural cardiac disease
This radiograph also reveals near complete opacification of the left hemithorax
with well-defined air bronchograms in the left lung. The infant’s endotracheal
tube is at the carina and needs to be pulled back. An umbilical venous catheter
is present with its tip in the right atrium and also needs to be pulled back. The
tip of the feeding tube is in the appropriate position.
Reference: Jones KL, Jones MC. Smith’s Recognizable Patterns of Human
Malformation. 7th edition. Elsevier. Philadelphia, 2013
Genetics & Skeletal System Question 3
A full-term infant born to a mother with type I diabetes mellitus is brought to
the NICU for evaluation because of immobility of the right arm. The following
film is obtained:


The infant is given some medication for pain management. Of the following, the
most preferred additional management of this infant is:
A. Evaluation for osteogenesis imperfecta
B. Immediate surgery for re-alignment
C. Immobilization of the arm
D. Provide supplemental calcium and vitamin D
Genetics & Skeletal System Answer 3
C. Immobilization of the arm
The radiograph of the infant in this vignette reveals an oblique fracture of the
right mid humeral diaphysis. This can occur if there is shoulder dystocia as a
result of a large for gestational age infant or if there is cephalopelvic
disproportion. Although the clavicle is the most common bone that can fracture
during delivery, the humerus is the most common long bone fracture in this
setting. Affected infants may present with irritability and arm immobility. If the
fracture is nondisplaced, infants may be asymptomatic. The examination may
reveal crepitus, pain, swelling, and decreased limb mobility.
Management of a newborn with a humeral fracture includes medication to
minimize pain, an orthopedics consultation, and immobilization of the arm for 2
to 4 weeks. Surgery for re-alignment is not necessary. If a newborn has a
humeral fracture without any risk factors, an evaluation for osteogenesis
imperfecta may need to be considered, especially if the fracture occurred in utero
or if there is evidence of additional fractures.
Reference: Rosenberg AA. Traumatic birth injury. NeoReviews.
2003;4:e270-e276
Genetics & Skeletal System Question 4
A term infant has a low oxygen requirement thought to be related to poor
inspiratory effort. A radiograph shown below reveals an incidental vertebral
abnormality evident by the green arrow:

Of the following, the most likely cause of this vertebral anomaly is:
A. An intrauterine vascular accident
B. An intrauterine vertebral fracture
C. Limited intrauterine mobility
D. Persistence of the notochord
Genetics & Skeletal System Answer 4
D. Persistence of the notochord
The radiograph of the infant in this vignette reveals a butterfly vertebral body
at the ninth thoracic level. In this finding, there is an absence of the midportion
of the vertebral body leading to a butterfly appearance of the vertebra. This
congenital anomaly results from a persistence of the notochordal tissue between
the lateral halves of the vertebral body. This persistence leads to failure of
fusion of the lateral halves of the vertebral body. Affected infants are usually
asymptomatic. Butterfly vertebra can be found in infants who have Pfeiffer
syndrome, Alagille syndrome, Jarcho-Levin syndrome or Crouzon syndrome.
Reference: Zuo K, Asenjo JF, Colmegna I. Butterfly vertebra. Arthr &
Rheum. 201365:195-6
Genetics & Skeletal System Question 5
A newborn with a skeletal abnormality has the following radiograph:

Of the following, the most likely characteristic or clinical finding in this infant
is:
A. Abnormality of the fibroblast growth factor 3 receptor gene
B. A cloverleaf skull
C. Excellent long-term survival
D. Intrauterine diagnosis not possible until the 3rd trimester
Genetics & Skeletal System Answer 5
A. Abnormality of the fibroblast growth factor 3 receptor gene
The radiograph of the newborn in this vignette shows short and curved femur
bones, often termed “telephone receivers”. This feature is found in infants with
thanatophoric dysplasia type I. Affected newborns also have a history of
polyhydramnios, macrocephaly, severely short limbs, a narrow thoracic cage,
and a protuberant abdomen. Newborns with type 2 thanatophoric dysplasia type
II similarly have short limbs and a narrow thoracic cage. However, infants with
type 2 have straight femurs and a cloverleaf skull. Both types of thanatophoric
dysplasia are caused by an abnormality of the fibroblast growth factor 3 receptor
gene. Of note, achondroplasia and hypochondroplasia are also associated with
defects in this gene.
Diagnosis of thanatophoric dysplasia is typically made during the 2nd
trimester by abnormal intrauterine sonographic skeletal findings. Survival
beyond the newborn period is rare in both types because of the associated
pulmonary hypoplasia and/or brainstem compression because of a narrow
foramen magnum.
Reference: Jones KL, Jones MC. Smith’s Recognizable Patterns of Human
Malformation. 7th edition. Elsevier. Philadelphia, 2013
Naveen NS, Murlimanju BM, Kumar V, Palakunta T, Jeeyar H.
Thanatophoric dysplasia: A rare entity. Oman Med J. 2011;26:196-197
Verma A, Mohan S, Kumar S. Thanatophoric dysplasia: Case report and
review of literature. J Clin Diagn Res. 2007;1:308-311
Genetics & Skeletal System Question 6
A neonatology fellow is teaching a group of pediatric residents about skeletal
abnormalities. He shows the residents the following film:

Of the following, the mother of this infant most likely has:


A. Diabetes mellitus type I
B. Graves’ disease
C. Hypoparathyroidism
D. Severe depression treated with a selective serotonin reuptake
inhibitor
Genetics & Skeletal System Answer 6
A. Diabetes mellitus type I
The radiographic in this vignette shows absence of the sacrum (see following
radiographic findings comparing radiographic findings of this infant with an
infant who has a normal sacrum)

This condition is sometimes referred to as caudal regression syndrome or caudal


dysplasia. It is a rare malformation that involves the legs, lumbar and coccygeal
vertebrae, and the affiliated spinal cord. It is associated with maternal diabetes
and vascular hypoperfusion. A complete evaluation of the infant in this vignette
is warranted as affected infants have associated respiratory, cardiac,
gastrointestinal, urinary, orthopedic, and neurologic systems. The remaining
options (Graves’ disease, hypoparathyroidism, and intrauterine exposure to a
selective serotonin reuptake inhibitor) have not been found to be associated with
sacral agenesis.
Reference: Boulas MM. Recognition of caudal regression syndrome. Adv
Neonatal Care. 2009;9:61-69
Genetics & Skeletal System Question 7
An obstetrical nurse contacts an infant’s pediatrician because of an
abnormality of the right arm. The pediatrician examines the infant and obtains
the following radiograph:

The pediatrician speaks with the family about the infant’s prognosis and
management.
Of the following, the most accurate statement about this infant’s diagnosis is:
A. African or African Americans are more affected than
Caucasians.
B. This infant has postaxial involvement.
C. This finding is a rare congenital anomaly.
D. This finding is most likely isolated.
Genetics & Skeletal System Answer 7
D. This finding is most likely isolated.
The radiograph of the infant in this vignette shows the presence of two
thumbs. There are 2 metacarpals in the expected location of the first metacarpal
with the first metacarpal having 1 tiny ossicle in the first proximal phalanx. The
next “first metacarpal” appears to have 2 ossicles. Both of these thumb-like
digits demonstrate abnormal morphology.
Polydactyly can be preaxial (occur on the side of the thumb or big toe),
postaxial (occur on the side of the little finger/small toe) or central (involve non-
border digits). Thus, the infant in this vignette has preaxial polydactyly.
Polydactyly is a common congenital anomaly with an incidence of preaxial
polydactyly reported to be as high as 1 in 3000 live births. Preaxial polydactyly
more commonly occurs in Caucasians compared to African or African American
populations and is usually an isolated finding (likely from a spontaneous
mutation). However, a triphalangeal thumb has an autosomal dominant mode of
inheritance. In contrast, postaxial polydactyly is more common in African or
African American populations (autosomal dominant pattern of inheritance, 1 in
145 live births, usually isolated with autosomal dominant inheritance pattern
with variable expression) than Caucasians (autosomal recessive transmission, 1
in 1400 live births, greater chance of being part of a syndrome). Some
syndromes associated with polydactyly include: Ellis-van Creveld syndrome,
trisomy 13, trisomy 21, and chrondroectodermal dysplasia.
Reference: Guo B, Lee SK, Paksima N. Polydactyly review. Bull Hosp Jt
Dis. 2013;71:17-23
Genetics & Skeletal System Question 8
A well appearing 1-week old infant born at 33 weeks’ gestation has a soft
murmur. The neonatology team orders a chest radiograph. The technician
mistakenly obtains a babygram, shown below:

Of the following, the testing that is LEAST likely to be helpful in the evaluation
of this patient is:
A. Abdominal ultrasonography
B. Echocardiography
C. Head imaging
D. Skeletal survey
Genetics & Skeletal System Answer 8
D. Skeletal survey
The radiograph of the infant in this vignette shows a gastric feeding tube
entering a right-sided intra-abdominal stomach bubble and a normally positioned
cardiac silhouette with a left apex and a slightly increased heart size. These
findings are consistent with heterotaxy syndrome. Heterotaxy can be divided
into bilateral right-sidedness (usually associated with asplenia) or bilateral left-
sidedness (usually associated with polysplenia syndrome). In contrast to infants
with situs inversus totalis (i.e., complete mirror image of the heart and
abdominal organs) who are unlikely to have other organ involvement, infants
with heterotaxy are at risk of having multi-organ abnormalities (40% to 70%)
including the following:
Structural heart disease
If asplenia, increased risk of total anomalous pulmonary venous return,
unbalanced atrioventricular septal defect, double outlet right ventricle,
pulmonary atresia or stenosis, absent coronary sinus, and bilateral “right
atrial-like” appendages.
If polysplenia, increase risk of inferior vena cava interruption with
azygous or hemiazygous continuation, atrioventricular septal defect, and
bilateral “left atrial-like” appendages.
Gastrointestinal abnormalities such as intestinal malrotation (70%) and
biliary atresia (10% if polysplenia)
Immunological abnormalities as a result of an absent or poorly functioning
spleen
Genitourinary abnormalities (26%) such as horseshoe kidney, renal
agenesis, hypoplastic or dysplastic kidneys, and ureteral abnormalities
Respiratory issues (sinopulmonary infections or bronchiectasis) related to
ciliary dysfunction
Neurological abnormalities such as hydrocephalus, agenesis of the corpus
callosum, holoprosencephaly, and meningomyelocele
Hematologic abnormalities leading to higher risk of thromboembolism
Venous anomalies such as extra hepatic portocaval communications
Thus, infants with heterotaxy require extensive diagnostic testing, which
includes abdominal ultrasonography echocardiography, and head imaging. As
skeletal abnormalities are unlikely, a skeletal survey is not required in the
evaluation of an infant diagnosed with heterotaxy.
Reference: Kothari SS. Non-cardiac issues in patients with heterotaxy
syndrome. Ann Pediatr Cardiol. 2014;7:187-192
Genetics & Skeletal System Question 9
An infant with skeletal dysplasia diagnosed prenatally has a postnatal radiograph
that shows an abnormality of the pelvic bones:

Of the following, the most likely mode of inheritance of this infant’s skeletal
dysplasia is:
A. Autosomal dominant
B. Autosomal recessive
C. X-linked dominant
D. X-linked recessive
Genetics & Skeletal System Answer 9
A. Autosomal dominant
The radiograph of the infant in this vignette shows rounded iliac wings with
small sacrosciatic notches and decreased acetabular angles leading to a
“tombstone” appearance. This radiograph also shows rounded metaphyseal
lucencies of the proximal femur.

These findings are evident in infants with achondroplasia. Achondroplasia is


the most common skeletal dysplasia. It has an autosomal dominant inheritance
pattern. Affected infants have the following radiographic findings:
Rhizomelia
Normal trunk length
“Trident hand” with short, wide, and cone-shaped phalanges
Craniofacial disproportion with large calvarium, small skull base
Short spinal pedicles with a decreased interpedicular distance in the lower
lumbar spine
Rounded iliac wings with decreased acetabular angles and small
sacrosciatic notches leading to a “tombstone” appearance
Wide and short tubular bones
Reference: Swarr DT, Sutton VR. Skeletal dysplasias in the newborn:
Diagnostic evaluation and developmental genetics. NeoReviews. 2010;11:e290-
e305
Genetics & Skeletal System Question 10
A female infant born at 24 weeks’ gestation has a clinical course complicated
by severe lung disease and surgical necrotizing enterocolitis associated with
small bowel syndrome. At 2 months the infant has a skeletal series because of a
concern for fractures.
Of the following, the LEAST likely radiograph that was obtained in this infant
is:
Genetics & Skeletal System Answer 10
C. Radiograph #3
The infant in this vignette most likely has a fracture as a result of severe
osteopenia because of a poor nutritional status associated with short gut
syndrome. The only radiograph that does not show evidence of osteopenia is
Radiograph #3. This film shows a fracture of the midshaft of the left clavicle
with some elevation of the proximal portion. The infant’s bones all have normal
density.
Radiograph #1 reveals an undisplaced fracture of the right humeral shaft and
severe osteopenia. Radiograph #2 shows an undisplaced right femoral fracture
with underlying osteopenia. Radiograph #4 reveals a left ulnar fracture with
decreased bone density consistent with osteopenia.
Genetics & Skeletal System Question 11
A radiologist reports an incidental finding on a newborn’s radiograph. The
infant does not have any dysmorphic features on clinical examination. The
neonatologist meets with the family and reports that this finding occurs in
approximately 5% to 8% of healthy newborns. In newborns diagnosed with
trisomy 21 syndrome, this finding can occur in 1/3 of affected patients.
Of the following, the radiograph that is most likely to represent this incidental
finding is:
Genetics & Skeletal System Answer 11
B. Radiograph #2
All of the radiographs in this vignette have an abnormal number of ribs. Ribs
develop from cartilaginous processes of thoracic vertebrae beginning at 9 weeks’
gestation. While the first seven rib pairs are directly attached to the sternum, the
next 3 rib pairs connect to the sternum indirectly by costal cartilages of the
higher rib with the last 2 pairs have no anterior attachment (often known as
floating ribs). Radiograph #2 reveals 11 pairs of ribs. As noted in this vignette,
this finding occurs in approximately 5% to 8% of healthy newborns. In
newborns diagnosed with trisomy 21 syndrome, this finding can occur in 1/3 of
affected patients. Eleven rib pairs are also found in infants with campomelic
dysplasia and trisomy 18. Radiograph #2 also reveals an endotracheal tube
overlying the mid trachea and an umbilical venous catheter with the tip at
thoracic vertebrae 9-10 near the inferior vena cava/right atrial junction. An
umbilical artery catheter is present with the tip at approximately thoracic
vertebrae 6-7. Granular interstitial opacities are seen that are most consistent
with surfactant deficiency.
Radiograph #1 shows a solitary left thirteenth rib. A feeding tube is noted
over the upper cervical/thoracic spine, which is likely caused by a catheter in an
esophageal pouch; clinical correlation is needed to confirm this diagnosis. A
higher number of ribs is more likely to be associated with other abnormalities,
such as trisomy 21 syndrome and the VACTERL association. Radiograph #3
shows 10 pairs of ribs, which is not a common finding in healthy newborns.
Although difficult to appreciate on this film, the infant was diagnosed with an
esophageal atresia.
Reference: Glass RBJ, Norton KI, Mitre SA, Kang E. Pediatric ribs: A
spectrum of abnormalities. RadioGraphics. 2002;22:87-104
Genetics & Skeletal System Question 12
The NICU team is contacted because a term newborn is having persistent
respiratory distress and cyanosis soon after delivery. The neonatology fellow
examines the infant and observes a large amount of oral secretions. A feeding
tube is placed and coils in the infant’s mouth. The team brings the infant to the
NICU for evaluation and obtains the radiograph on the following page:

Of the following, the most likely additional finding in this infant is(are):
A. Anal atresia
B. Choanal atresia
C. Colobomas
D. Micropenis
Genetics & Skeletal System Answer 12
A. Anal atresia
The radiograph of the infant in this vignette demonstrates coiling of the
feeding tube and a gasless abdomen suggesting the diagnosis of esophageal
atresia without a distal tracheoesophageal fistula. In addition, there are several
upper thoracic vertebral anomalies with multiple hemivertebrae. There are rib
abnormalities as well with thin and thick ribs. The radiograph also shows that the
infant has a mild scoliosis of the thoracic spine concaved to the left. vertebral
anomalies. These 2 abnormalities can be found in infants with VACTERL
association, which consists of the following features:
V-vertebral anomalies (80%) such as hemivertebrae, scoliosis, and sacral
dysgenesis
A-anal atresia (55%)
C-cardiac defects (75%) most often associated with septal defects
TE-tracheoesophageal abnormalities (70%)
R-renal anomalies (50%) such as renal agenesis, ureteropelvic junction
obstruction, and severe reflux
L-limb anomalies (70%) that typically involve the arms more than the legs
and include radial aplasia, deviation of the hand, absent or hypoplastic
thumb, and preaxial polydactyly
Structural heart disease (75%)
Thus, of the options provided, the most likely additional finding in the infant in
this vignette is anal atresia. Affected infants typically have nondysmorphic
facies and a normal growth pattern.
The remaining options are found in infants with CHARGE syndrome. The
prevalence of CHARGE syndrome is 1 per 10-15,000 and it has an autosomal
dominant mode of inheritance. Affected individuals may have the following
features:
C-colobomas (80% to 90%) with retinal colobomas more common than
iris colobomas
H-heart defects (75% to 85%) typically conotruncal anomalies or aortic
arch abnormalities
A-atresia choanae (50% to 60%) that can be partial or complete, unilateral
or bilateral
R-retarded growth and development during childhood (of note, affected
newborns appropriate size)
G-genital hypoplasia (50% to 60%) such as microphallus, hypospadias,
cryptorchidism, chordee, and bifid scrotum in males and hypoplastic labia
or clitoris in females
E-ear anomalies (90%) that can involve the outer, middle or inner ear
In addition, renal anomalies have been found in 25% to 40% of affected
patients. Finally, cranial nerve anomalies are also common, occurring in 75% to
95% and have recently been added to the major diagnostic criteria.
Reference: Kaplan J, Hudgins L. Neonatal presentations of CHARGE
syndrome and VATER/VACTERL association. NeoReviews. 2008;9:e299-e304
IX. OPHTHALMOLOGY
Ophthalmology Question 1
Put the images in order from early to late progression of retinopathy of
prematurity (ROP).

A. 1, 2, 3
B. 2, 3, 1
C. 2, 1, 3
D. 3, 2, 1
Ophthalmology Answer 1
C. 2, 1, 3
The first stage of ROP is characterized by a line separating the vascular and
avascular retina and mild neovascularization at the line. A more prominent ridge
with increased neovascularization defines the second stage. Stage 3 ROP is
defined as the presence of fibrovascular proliferation. Not pictured are Stage 4
characterized by the fibrovascular material attaching to the zonules and lens
resulting in a partial retinal detachment and Stage 5, total retinal detachment.
References: http://ropmalpractice.com/wp-content/uploads/2011/03/ROP-
Chart-207.jpg and http://www.rostimes.com

1. Avascular Zone
2. Vascular Zone
3. Demarcation Line
4. Ridge
5. Fibrovascular Proliferation
6. Neovascularization
Ophthalmology Question 2
During a newborn examination, a medical student observes the following
ocular findings:

All of the following statements are true, EXCEPT:


A. In the left panel, all layers of the eye can be affected
B. In the right panel, if the lesion were in the lower lid, this can be associated
with Treacher-Collins syndrome
C. Lower lid defects are more common than upper lid defects
D. Upper eyelid defects are associated with Goldenhar syndrome
Ophthalmology Answer 2
C. Lower lid defects are more common than upper lid defects
The images in this vignette reveal colobomata at different locations. The
panel on the left shows an iris coloboma and the panel on the right shows an
upper lid coloboma. In ocular colobomata, all layers of the eye can be affected
and the degree of involved regions will define ocular outcomes. Upper eyelid
colobomata are more common than lower eyelid colobomata and are often
isolated except when associated with Goldenhar syndrome. Lower eyelid
colobomata are associated with Treacher-Collins syndrome. There is a long
differential when considering the origin of a coloboma and familial (inheritance),
syndromes, and chromosomal etiologies must all be considered.
Reference: Onwochei BC. Ocular colobomata. Surv Ophthalmol. 2000 Nov-
Dec;45(3):175-94.
Ophthalmology Question 3
A neonatology fellow is teaching a pediatric resident about the stages of
retinopathy of prematurity.

Retinopathy in Zone _____ is most concerning.


A. I
B. II
C. III
Ophthalmology Answer 3
A. I
In addition to staging, the severity of ROP is also defined by the Zone or
region of the retina that demonstrates disease. Zone 1 is the center of the retina
including the optic nerve and macular center. Zone 3 is in the periphery; Zone II
is between Zones 1 and 3. The most severe type of ROP occurs in Zone 1 as it
interferes with early retinal vascularization and contains the vital structures of
the optic nerve and macular center. The presence of plus disease is the third
variable in assigning ROP severity.
Ophthalmology Question 4
On examination of an infant born at 31 weeks’ gestation, the Fellow notices
white linear densities and a slightly unequal pupil shape in one eye.

Of the following, the most likely diagnosis is:


A. Early cataract formation
B. Incomplete coloboma
C. Normal variant
D. Persistent pupillary membrane
Ophthalmology Answer 4
D. Persistent pupillary membrane
This schematic depicts a persistent pupillary membrane (PPM). This
membrane is present during fetal development providing a rich blood supply to
the eye. It typically regresses in the sixth month of gestation. The failure of
normal regression results in PPM. The remnant bands can be obstructive to
normal vision and lead to obstructive amblyopia. Another associated ocular
outcome is the development of cataracts.
References: Kraus CL, Lueder GT. Clinical characteristics and surgical
approach to visually significant persistent pupillary membranes. J AAPOS. 2014
Dec;18(6):596-9 and Shlamovitz GZ, Reardon PC. Images in clinical medicine.
Persistent pupillary strands and membranes. N Engl J Med. 2010 Apr
8;362(14):1325.
X. LINES & TUBES
Lines & Tubes Question 1
Match the most likely position of the umbilical catheter tip (A-E) with the
appropriate radiograph (#1 - #6). Each radiograph option can only be used
once.
A. Umbilical arterial catheter tip in the aorta (thoracic level)
B. Umbilical arterial catheter tip in the aorta (lumbar level)
C. Umbilical venous catheter tip in a left hepatic vein
D. Umbilical venous catheter tip in a pulmonary vein
E. Umbilical venous catheter tip in a venous branch in the right side of the
liver
F. Umbilical venous catheter tip in the left atrium
Lines & Tubes Answer 1
A. radiograph #2; B. radiograph #3; C. radiograph #1; D. radiograph #6; E.
radiograph #4; F. radiograph #5
When placing catheters in the umbilical cord, the location in the artery vs
vein can be distinguished by the course that the catheter follows after it enters
the neonate’s body. The umbilical venous catheter enters and courses superiorly
and directly to the inferior vena cava via the ductus arteriosus. In contrast, an
umbilical arterial catheter courses inferiorly and then moves superiorly to merge
with the internal iliac artery. The anatomy of the fetal circulation is shown in the
diagram below.

The umbilical arterial line tip in radiograph #2 is in the aorta at the level of
the thoracic vertebrae 4-5 and needs to be pulled back. The desired location of
the umbilical arterial line tip is between the 6th and 9th thoracic vertebrae to avoid
the vessels originating from the aorta below T9 (celiac, mesenteric and renal
arteries) and the aortic arch and patent ductus arteriosus above T6. The umbilical
venous catheter tip in radiograph #2 appears to project over the liver. A cross-
table lateral view will help to confirm the position of the venous catheter; if it is
located within the liver, it will need to be removed.
The umbilical catheter in radiograph #3 is an arterial line because it enters
inferiorly before moving superiorly and passing through the internal iliac artery
into the common iliac artery and then the aorta. The catheter tip is at the
superior border of the 2nd lumbar vertebrae in the aorta, slightly below the renal
arteries. This position tip is too low and needs to be removed to avoid
complications. The appropriate low position for an umbilical arterial catheter tip
is below the bottom of the 3rd lumbar vertebrae to avoid the renal arteries, which
are at the 1st lumbar vertebrae.
The tip of the umbilical venous catheter in radiograph #1 is in a left hepatic
vein. Because of this inappropriate position, this catheter needs to be removed.
There are 2 umbilical lines shown in radiograph #6. The umbilical arterial
line tip is at the aortic arch while the umbilical venous catheter crosses the
foramen ovale and terminates in the pulmonary vein. Both lines need to be
pulled back and a follow-up radiograph is needed to confirm appropriate tip
placement.
Radiograph #4 reveals an umbilical venous catheter line with the tip in the
right lobe of the liver, likely within a branch of the right portal vein. Because of
this inappropriate distal position, this catheter needs to be removed.
The umbilical catheter line in radiograph #5 is in an umbilical vein as the
entry moves superiorly from the umbilical vein, through the ductus venosus, and
into the inferior vena cava. Following, this infant’s umbilical venous catheter
enters the right atrium and then crosses the foramen ovale with the tip in the left
atrium. Because the desired tip location of the umbilical venous line is in the
distal inferior cava, this infant’s catheter needs to be pulled back.
Reference: Fletcher MA. Physical Diagnosis in Neonatology. Lippincott
Williams & Wilkins;1997
Lines & Tubes Question 2
A 1-hour old male infant born at 28 weeks’ gestation is placed on CPAP with
a PEEP of 6 cm H2O and 27% FiO2. The pediatric intern and neonatology
fellow plan to place an umbilical venous catheter. However the fellow is called
to assess an infant in respiratory distress and needs to leave the intern alone to
complete the procedure. The intern places the umbilical catheter and is
uncertain if he placed the line in the umbilical vein or artery. To assess
placement, the intern obtains the following chest radiograph:

Of the following, the most likely tip position of this umbilical line is the:
A. Aorta
B. Ductus venosus
C. Inferior vena cava
D. Right atrium
Lines & Tubes Answer 2
D. Right atrium
The straight path of entry of the umbilical catheter shown on this infant’s film
is consistent with entry into the umbilical vein. The tip of the line is in the
inferior right atrium. Because this is not an ideal location, the intern will need to
pull back the line.
Lines & Tubes Question 3
A male infant born at 28 weeks’ gestation with normal anatomy has severe
respiratory distress requiring intubation and surfactant. The pediatric resident
places umbilical venous and arterial lines and confirms placement by an AP
radiograph, shown below:

A lateral abdominal radiograph of this infant would show that the “Line A” most
likely:
A. Enters and curves inferiorly and then superiorly
B. Has its tip in the left atrium
C. Is located anterior to the other posteriorly located line
D. Is located in the ideal position
Lines & Tubes Answer 3
C. Is located anterior to the other posteriorly located line
The line designated as A most likely is the umbilical venous line as a catheter
in an infant’s umbilical vein should be located to the right of midline on an AP
radiographic view. In contrast, an umbilical arterial line usually has its tip at or
slightly to the left of midline. While the umbilical arterial line entrance into the
infant’s body initially curves inferiorly and then moves superiorly as it enters the
internal iliac artery, the umbilical venous catheter enters straight and superiorly.
Because Line A is most likely the umbilical venous catheter, a lateral abdominal
film will show the umbilical venous catheter (Line A) to be located anterior to
the umbilical artery catheter (Line B). The umbilical venous catheter tip is most
likely at the junction of the superior vena cava and right atrium and needs to be
pulled back prior to initiating use. The umbilical artery catheter tip is in an
appropriate position within the aorta.
Of note, this film also shows bilateral diffuse granular opacities, which are
nearly confluent, especially in the right lung. The endotracheal tube tip is just
above the carina
Reference: Kirpalani H, Epelman M, Mernah JR. Imaging of the Newborn.
nd
2 edition. Cambridge University Press, 2012
Lines & Tubes Question 4
A 6-day old infant born at 28 weeks’ gestation has had some feeding
intolerance requiring a slow feeding advancement. As a result, the neonatal
nurse practitioner inserts a peripherally inserted central catheter (PICC) into the
infant’s right arm to provide continued total parenteral nutrition. Before
removing the infant’s umbilical venous catheter line, she obtains a radiograph to
assess the position of the new line.

The nurse practitioner’s most likely next step is to:


A. Begin using the line as the position is appropriate
B. Obtain a lateral radiograph to determine the precise location of the PICC
C. Pull the PICC line back 2 cm and obtain another radiograph
D. Remove the PICC line
Lines & Tubes Answer 4
C. Pull the PICC line back 2 cm and obtain another radiograph
The chest radiograph of this infant shows the right arm PICC line tip in the
right atrium. The ideal tip location of upper body insertions of PICC lines is the
lower 1/2 to 1/3 of the superior vena cava. Because the tip of the PICC line is
within the right atrium, the infant is at risk of developing a(n) arrhythmia,
intracardiac thrombus, pericardial effusion with possible cardiac tamponade,
cardiac perforation, or pleural effusion. To prevent these potential
complications, the nurse practitioner will need to maintain sterility and pull the
PICC line back approximately 2 cm. Prior to PICC use, a repeat film is needed
to confirm appropriate tip position.
Of note, the chest radiograph of this infant also shows that the distal tip of the
umbilical venous line catheter is in the inferior vena cava. There is bilateral
ground-glass opacification of the lungs, and the heart size and pulmonary
vascularity are within normal limits. A vertical line projecting over the right
hemithorax is most likely a skin fold.
Lines & Tubes Question 5
A 2-day old infant born at 24 weeks’ gestation has an acute change in
respiratory status. The neonatology team obtains an arterial blood gas, which
shows a severe respiratory acidosis, prompting an increased in ventilator
settings. The infant’s radiograph is shown below:

The line(s) or tube(s) shown in this film that are in an inadequate position is
(are):
A. Endotracheal tube only
B. Nasogastric tube and endotracheal tube
C. Umbilical arterial catheter and endotracheal tube
D. Umbilical venous catheter and endotracheal tube
Lines & Tubes Answer 5
C. Umbilical arterial catheter and endotracheal tube
The radiograph of this infant shows that the tip of the endotracheal tube is
positioned at the thoracic inlet. To avoid an unplanned extubation, the
endotracheal tube should be positioned within the trachea. In addition, the distal
end of the umbilical arterial catheter is coiled within the abdominal aorta and
needs to be removed.
The tips of the umbilical venous catheter (located at the inferior cavoatrial
junction) and nasogastric tube (located at the fundus of the stomach) shown on
this infant’s radiograph are in the appropriate position. Of note, this radiograph
also shows near complete opacification of both lungs and a moderate-sized right
pleural effusion. There is moderate total body wall edema and the heart size
cannot be discerned.
Lines & Tubes Question 6
A full-term infant with severe perinatal depression is receiving therapeutic
hypothermia. An acute change in respiratory status leads the neonatologist to
order a radiograph, shown below:

Of the following, the most appropriately positioned line or tube shown on this
radiograph is the:
A. Endotracheal tube
B. Esophageal probe
C. Umbilical arterial catheter
D. Umbilical venous catheter
Lines & Tubes Answer 6
D. Umbilical venous catheter
The radiograph of the infant in this vignette shows that the umbilical venous
catheter is in the appropriate position with the distal tip slightly above the level
of the diaphragm between the 8th and 9th thoracic vertebral bodies.
In contrast, this infant’s umbilical arterial catheter tip is high because it is
between the 5th and 6th thoracic vertebral bodies and the desired umbilical arterial
line tip should rest between the 6th and 9th thoracic vertebral bodies. At this
current location above thoracic vertebrae 6, the line might enter the aortic arch
and patent ductus arteriosus.
The endotracheal tube of this infant has its distal tip past the carina in the
direction of the right mainstem bronchus. This tube needs to be pulled back to
maintain effective bilateral air entry and avoid left lung atelectasis.
The bulbous tip of this infant’s temperature probe is positioned high in the
esophagus. The ideal position for this esophageal sensor is in the distal third of
the esophagus.
In addition to these lines and tubes, the chest radiograph shows an external
monitor lead crossing horizontally over the infant.
Lines & Tubes Question 7
Match the most likely infant scenario (A-D) with the appropriate radiograph
(#1 - #4). Each radiograph option can only be used once.
A.Chronic ventilator dependence
B.Full-term infant with a pre-ductal oxygen saturation of 65% at 3 minutes of
age without respiratory distress
C.Severe feeding dysmaturity in a former 24 week infant with a
postmenstrual age of 46 weeks’ gestation
D. Short gut syndrome following surgical necrotizing enterocolitis
Lines & Tubes Answer 7
A. radiograph #3; B. radiograph #4; C. radiograph #2; D. radiograph #1
The most likely radiograph in an infant with chronic ventilator dependence is
radiograph #3 as it shows an infant with a tracheostomy tube. The reason for
this ventilator dependence is not evident on this film as the lung fields look well
expanded without severe disease. Potential causes of ventilator dependence
include: severe chronic lung disease of prematurity, neurological disorders
leading to chronic hypoventilation, and severe persistent apnea.
A normal pre-ductal oxygen saturation in a full-term infant at 3 minutes of
age is 70% to 75%. If a full-term infant has a pre-ductal oxygen saturation of
65% at 3 minutes of age, the most appropriate response of the delivery room
team is to administer blow by oxygen; this oxygen tubing near the infant’s
mouth is evident in radiograph #4.
The most likely radiograph of an infant with severe feeding dysmaturity is
radiograph #2 as it shows an infant with a gastrostomy tube. If an infant is likely
to feed orally within a short time, home nasogastric feedings may be
recommended instead of a gastrostomy tube.
An infant with short gut syndrome following surgical necrotizing enterocolitis
has a high likelihood of being dependent on total parenteral nutrition.
Radiograph #1 shows a right internal jugular line that ends in the inferior aspect
of the right atrium. This line can be used to provide an infant with chronic
parenteral nutrition. Of note, this film shows some perihilar interstitial
thickening with some mild hazy opacities throughout the lungs. The
endotracheal tube tip is at the carina and needs to be repositioned.
Lines & Tubes Question 8
A female infant born at 24 weeks’ gestation is now 1 month old. Her clinical
course has been complicated by a patent ductus arteriosus requiring surgical
ligation, medical necrotizing enterocolitis, and ventilator dependence. A new
PICC line is placed in the infant’s left arm by the neonatology fellow to ensure
that the infant receives appropriate caloric intake while awaiting tolerance of full
feedings. The post-procedure film demonstrates the following:

Of the following, the most likely position of the tip of this PICC line is the:
A. Descending aorta
B. Inferior vena cava
C. Right atrium
D. Right ventricle
Lines & Tubes Answer 8
B. Inferior vena cava
The infant’s supine radiograph shows that the left arm PICC line is traversing
the superior vena cava and right atrium with the distal tip positioned in the
inferior vena cava. The film also shows a feeding tube located in the infant’s
left-sided stomach, and a clip over the ductus arteriosus. After viewing this film,
the fellow pulls back the PICC and obtains a follow-up film to ensure that the tip
is located in the lower 1/2 to 1/3 of the superior vena cava.
Lines & Tubes Question 9
After a pediatric resident places umbilical venous and arterial lines in a 1-
hour old newborn born at 28 weeks’ gestation, he orders a chest radiograph,
shown below:

The pediatric resident discusses the endotracheal tube placement with the fellow
and the respiratory therapist who had helped position the infant during the film.
The head position that would most likely lead to lower placement of this infant’s
endotracheal tube is:
A. Flexed neck positioning
B. Hyperextended neck positioning
C. Prone positioning
D. Supine positioning
Lines & Tubes Answer 9
A. Flexed neck positioning
Neck position (i.e., extension vs flexion) can impact the position of the tip of
the endotracheal tube. If an infant’s neck is flexed, the endotracheal tube will
give a move lower while an infant with a hyperextended neck will cause the
endotracheal tube to move higher. Knowledge of an infant’s neck position at the
time of a radiograph is important to determine whether an action is needed to
reposition an infant’s endotracheal tube. Ideally, an infant’s neck should be
placed in a neutral position at the time of a radiograph.

You might also like